Ambulatory - Int Med Case File Questions

अब Quizwiz के साथ अपने होमवर्क और परीक्षाओं को एस करें!

Target LDL in diabetic patient

100 mg/dL (ACTUALLY <70)

Disorders of secondary hemostasis

(coagulation factor deficiencies such as hemophilia) usually are characterized by the development of superficial ecchymoses as well as deep hematomas and hemarthroses.

Definition of impaired systolic function

(ejection fraction <40%)

Disorders of primary hemostasis - sxs

(thrombocytopenia or von Willebrand disease) are characterized by mucosal bleeding and the appearance of petechiae or superficial ecchymoses.

Top 2 most common types of dementia

Alzheimer disease is the most common type of dementia, followed by multi-infarct (vascular) dementia.

22.4 A 22-year-old man presents with complaints of low back pain for 3 to 4 months and stiffness of the lumbar area, which worsen with inactivity. He reports difficulty in getting out of bed in the morning and may have to roll out sideways, trying not to flex or rotate the spine to minimize pain. A lumbosacral (LS) spine x-ray film would most likely show which of the following? A. Degenerative joint disease with spur formation B. Sacroiliitis with increased sclerosis around the sacroiliac joints C. Vertebral body destruction with wedge fractures D. Osteoporosis with compression fractures of L3-L5 E. Diffuse osteonecrosis of the LS spine

B. A young man is not likely to have osteoporosis, osteoarthritis, or compression fractures. His morning stiffness, which worsens with rest, suggests an inflammatory arthritis, such as ankylosing spondylitis, which would include sacroiliitis with increased sclerosis around the sacroiliac joints. reactive arthritis = type of spondyloarthropathy.

54.1 Which of the following asymptomatic patients would most benefit from treatment of the finding of more than 105 CFU/mL of Escherichia coli on urine culture? A. A 23-year-old asymptomatic sexually active woman B. A 33-year-old asymptomatic pregnant woman C. A 53-year-old asymptomatic diabetic woman D. A 73-year-old asymptomatic woman in a nursing home

B. All of these patients are asymptomatic, and no benefit from treatment in terms of reduction in symptomatic UTIs or hospitalization has been shown for any of the other cases mentioned, except for pregnancy. Treatment is undertaken to prevent upper tract infection, preterm delivery, and possible fetal loss.

49.4 Which of the following are commonly seen in brain imaging of patients with Alzheimer disease? A. Normal cerebral ventricles and atrophic brain tissue B. Enlarged cerebral ventricles and atrophic brain tissue C. Enlarged cerebral ventricles and no atrophy of brain tissue D. Normal cerebral ventricles and normal brain tissue, acetylcholine deficiency

B. Alzheimer disease typically has enlarged cerebral ventricles and brain atrophy, whereas normal pressure hydrocephalus has enlarged brain ventricles without brain atrophy.

36.1 On routine blood work performed for a life insurance application, a 48-year-old premenopausal woman was found to have a calcium level of 12 mg/dL (normal = 8.8-10.4 mg/dL) and a phosphate level of 2 mg/dL (normal = 3.0-4.5 mg/dL). She is not anemic and has no symptoms. Her medical history is significant for osteoporosis, discovered on a dual-energy x-ray absorptiometry (DEXA) scan performed last year. Which of the following is the most likely cause of her hypercalcemia? A. Multiple myeloma B. Parathyroid adenoma C. Familial hypocalciuric hypercalcemia D. Sarcoidosis E. Undiagnosed breast cancer

B. An asymptomatic, most likely chronically elevated calcium level is most likely caused by primary hyperparathyroidism due to a parathyroid adenoma. The hypercalcemia is presumed to be chronic because she has osteoporosis and is premenopausal.

42.1 A patient comes in for a fasting plasma glucose test. On two separate occasions, the result has been 115 mg/dL and 120 mg/dL. Which of the following is the most appropriate next step? A. Reassurance that these are normal blood sugars. B. Recommend weight loss, an ADA diet, and exercise. C. Diagnose diabetes mellitus and start on a sulfonylurea. D. Recommend cardiac stress testing. E. Obtain stat arterial blood gas and serum ketone levels.

B. By diagnostic criteria, this patient falls into the definition of impaired fasting glucose. Although she does not yet meet the criteria for diabetes, she is at greater risk for developing diabetes in the future and for macrovascular disease. Intensive lifestyle changes (diet and exercise for 30 minutes per day, 5 days per week) can reduce or delay the development of diabetes. Patients should be monitored annually to screen for progression to diabetes. American Diabetes Association recommends metformin should be considered if prediabetes especially if BMI >35 kg/m2, <60, and woman with a history of gestational diabetes. This

25.1 A 25-year-old man with a history of a duodenal ulcer is noted to have a hemoglobin level of 10 g/dL. He does not report any visible GI blood loss. Which of the following most likely will be seen on laboratory investigation? A. Reticulocyte count of 4% B. Elevated total iron-binding capacity C. Normal serum ferritin D. Mean corpuscular volume of 105 fL

B. Chronic gastrointestinal blood loss leads to low ferritin levels reflecting diminished iron stores, elevated TIBC, and low iron saturation. There is a microcytic anemia (low MCV) with a low reticulocyte count. **The reticulocyte count would be elevated with acute blood loss, but the patient has not experienced this.

10.2 An 80-year-old woman without a history of hypertension undergoes surgery for a hip fracture. Her blood pressure on postoperative day 1 is 178/110 mm Hg. She is asymptomatic except for hip pain. Which of the following is the best next step? A. Transfer the patient to the intensive care unit, obtain cardiac enzyme levels, and lower the blood pressures to the 140/90 mm Hg range. B. Control the pain and monitor the blood pressure. C. Start the patient on a beta-blocker and monitor the blood pressure. D. Restrict visitors and turn down television, alarms, and other noise.

B. Elevated blood pressures without symptoms may occur acutely after surgery, particularly as a consequence of postoperative pain. Blood pressure medications are usually not indicated, but rather, pain control is the primary treatment. Lowering of blood pressure can lead to orthostatic hypotension when the patient gets out of bed.

9.4 A 45-year-old man with type 2 diabetes is noted to have blood pressures of 145/90 and 150/96 mm Hg on two separate occasions. Which of the following is the best initial therapy for this patient? A. Hydrochlorothiazide B. ACE inhibitor C. Beta-blocker D. Beta-blocker and hydrochlorothiazide

B. For diabetics, in general, the antihypertensive agent of choice is the ACE inhibitor. If the blood pressure is uncontrolled, then a thiazide diuretic may be added. The patient will have survival advantage with the ACE inhibitor.

56.3 Painful, swollen metatarsophalangeal great toe (unilateral) with redness and warmth after eating a steak and shrimp dinner in a 45-year-old man A. Gonococcal arthritis B. Gout C. Pseudogout D. Osteoarthritis E. Rheumatoid arthritis F. Systemic lupus erythematosus

B. Gouty arthritis often affects the fi rst metatarsophalangeal joint and can be precipitated by various foods or alcohol.

13.2 A 56-year-old man has brittle diabetes (difficult to control with widely fluctuating blood sugars), tan skin, and a family history of cirrhosis. Select the cause (A-G) that is probably responsible for the patient's presentation. A. Wilson disease B. Hematochromatosis C. Primary biliary cirrhosis D. Sclerosing cholangitis E. Autoimmune hepatitis F. Alcohol-induced hepatitis G. Viral hepatitis

B. Hemochromatosis is a genetic disorder of iron metabolism. Progressive iron overload leads to organ destruction. Diabetes mellitus, cirrhosis of the liver, hypogonadotropic hypogonadism, arthropathy, and cardiomyopathy are among the more common end-stage developments. Skin deposition of iron leads to "bronzing" of the skin, which could be mistaken for a tan. Diagnosis is made early in the course of disease by demonstrating elevated iron stores but can be made through liver biopsy with iron stains. Genetic testing is available. Therapy involves phlebotomy to remove excess iron stores.

2.2 In the United States, which of the following is most likely to have caused the congestive heart failure (72 yo man w/ exertional dyspnea, orthopnea and PND w/ physical signs of overload - likely secondary to Aortic Stenosis) in the patient described in Question 2.1? A. Diabetes B. Atherosclerosis C. Alcohol D. Rheumatic heart disease

B. In the United States, the most common cause of CHF associated with impaired systolic function is ischemic cardiomyopathy due to coronary atherosclerosis.

25.2 A 22-year-old woman is pregnant and at 14-week gestation. Her hemoglobin level is 9 g/dL. She asks why she could have iron deficiency when she is no longer menstruating. Which of the following is the best explanation? A. Occult gastrointestinal blood loss B. Expanded blood volume and transport to the fetus C. Hemolysis D. Iron losses as a result of relative alkalosis of pregnancy

B. Iron deficiency occurs in pregnancy as a result of the expanded blood volume and active transport of iron to the fetus.

25.3 A 35-year-old man has undertaken a self-imposed diet for 3 months. He previously had been healthy but now complains of fatigue. His hemoglobin level is 10 g/dL, and his MCV is 105 fL. Which of the following is the most likely etiology of his anemia? A. Iron deficiency B. Folate deficiency C. Vitamin B12 deficiency D. Thalassemia E. Sideroblastic anemia

B. Macrocytic anemia is usually a result of folate or vitamin B12 deficiency. Because vitamin B12 stores last for nearly 10 years, a dietary change of several months would more likely cause folate deficiency. Also vitamin B12 deficiency can lead to neurologic symptoms. Folate is found in green leafy vegetables.

55.2 Which of the following findings is more consistent with an osmotic, rather than a secretory, diarrhea? A. The diarrhea persists despite a 48-hour fast. B. Stool osmolality = 290 mOsm, stool Na = 95 mOsm, stool K = 15 mOsm. C. Diarrhea is large volume and watery, and is accompanied by paroxysms of flushing and wheezing. D. Profuse, painless "rice-water" stool in a patient in a cholera-endemic area.

B. Normal stool osmolality is equal to plasma, about 290 mOsm. In secretory diarrhea, most of the osmotically active particles are electrolytes, and can be calculated as 2 × [Na + K]. The size of osmotic gap (the difference between calculated and directly measured osmolality) is equivalent to the concentration of the poorly absorbed unmeasured solute in the fecal water. This patient has a stool osmotic gap of 70 (gap >50 is indicative of osmotic diarrhea). Answers C and D are suggestive of carcinoid syndrome and cholera infection, respectively, both causes of secretory diarrhea. osmotic = ingestion of large amounts of poorly absorbed, osmotically active solute taht draws water into the intestinal lumen

9.2 A 45-year-old man is diagnosed with idiopathic hypertension based on two blood pressures of 150/100 and 156/102 mm Hg. Which of the following would most likely provide prognostic information regarding this patient? A. Vascular biopsy B. End-organ effects from hypertension, such as left ventricular hypertrophy C. Patient's enrollment in a clinical trial D. Measurement of serum homocysteine levels

B. Prognosis in hypertension depends on the patient's other cardiovascular risks and observed end-organ effects from the hypertension.

4.1 A 42-year-old overweight but otherwise healthy woman presents with sudden onset of right-upper abdominal colicky pain 45 minutes after a meal of fried chicken. The pain is associated with nausea and vomiting, and any attempt to eat since has caused increased pain. Which of the following is the most likely cause? A. Gastric ulcer B. Cholelithiasis C. Duodenal ulcer D. Acute hepatitis

B. Right-upper abdominal pain of acute onset that occurs after ingestion of a fatty meal and is associated with nausea and vomiting is most suggestive of biliary colic as a result of gallstones. -Duodenal ulcer pain is likely to be diminished with food -gastric ulcer pain is not likely to have acute severe onset. -Acute hepatitis is more likely to produce dull ache and tenderness.

22.2 A 65-year-old man with a history of chronic hypertension, diabetes mellitus, and degenerative joint disease presents with acute onset of severe pain of the metatarsophalangeal (MTP) joint and swelling of the left first toe. Physical examination shows exquisite tenderness of the joint, with swelling, warmth, and erythema. The patient has no history of trauma or other significant medical problems. Synovial fluid analysis and aspiration are most likely to show which of the following? A. Hemorrhagic fluid B. Needle-shaped, negatively birefringent crystals C. Gram-negative organisms D. Noninflammatory fluid E. Rhomboidal, positively birefringent crystals

B. The involvement of the great toe is most likely gout, and the synovial fluid is likely to show needle-shaped, negatively birefringent crystals.

24.2 A 32-year-old woman from Nigeria presents with a 12-week history of persistent lower lumbar back pain, associated with a low-grade fever and night sweats. She denies any extremity weakness or HIV (human immunodeficiency virus) risk factors. Her examination is normal except for point tenderness over the spinous processes of L4-L5. Which of the following is the most likely diagnosis? A. Staphylococcus aureus osteomyelitis B. Tuberculous osteomyelitis C. Given her age, idiopathic low back pain D. Metastatic breast cancer E. Multiple myeloma

B. The patient's country of origin, the chronic and slowly progressive nature of the pain in association with fever, and night sweats are highly suggestive of tuberculous osteomyelitis of the spine, or Pott disease. Bacterial osteomyelitis presents more acutely, often with high, spiking fevers. Metastatic breast cancer and multiple myeloma are extremely rare in this age group. The fevers, night sweats, and persistent and progressive nature of her back pain make a musculoskeletal cause unlikely.

41.1 A 38-year-old man with a 12 pack of beer per day alcohol history presents with jaundice, ascites, and dark urine. His laboratory results are: AST 350 U/mL,, ALT 150 U/mL, alkaline phosphatase 120 U/mL, total bilirubin 25 mg/dL, direct bilirubin 12 mg/dL, and albumin 2.1 g/dL. A. Hemolysis B. Alcoholic hepatitis C. Gilbert disease D. Pancreatic cancer E. Gallstones F. Primary sclerosing cholangitis

B. The patient's laboratory results show a conjugated hyperbilirubinemia with evidence of hepatocellular disease (hypoalbuminemia, ascites). The AST and ALT levels show the 2:1 ratio consistent with alcohol-related liver disease.

11.3 In a suicide attempt, an 18-year-old adolescent female took 4 g of acetaminophen, approximately 8 hours previously. Her acetaminophen level is 30 μg/mL. Which of the following is the best next step to be performed for this patient? A. Immediately start N-acetylcysteine B. Observation C. Alkalinize the urine D. Administer intravenous activated charcoal

B. The serum acetaminophen level of 30 μg/mL, with last ingestion 8 hours previously, is plotted on the nomogram and falls below the "danger zone" of possible hepatic injury. Thus, this patient should be observed. Sometimes, patients will take more than one medication so that serum and/or urine drug testing may be worthwhile. Gastrointestinal activated charcoal, not intravenous charcoal, is used for other ingestions

49.2 A 74-year-old man was noted to have excellent cognitive and motor skill 12 months ago. His wife noted that 6 months ago his function deteriorated noticeably, and 2 months ago another level of deterioration was noted. Which of the following is most likely to reveal the etiology of his functional decline? A. HIV antibody test B. Magnetic resonance imaging of the brain C. Cerebrospinal fluid (CSF) Venereal Disease Research Laboratory (VDRL) test D. Serum thyroid-stimulating hormone

B. The stepwise decline in function is typical for multi-infarct dementia, diagnosed by viewing multiple areas of the brain infarct.

33.1 A 25-year-old man presents to your office complaining of left knee and right great toe pain, which started 1 week ago and has not responded to over-the counter pain relievers. He also has felt feverish and achy, has dysuria, and has developed an eye infection. Approximately 1 month ago, he was seen at an outside clinic and treated for syphilis. On examination, he is afebrile, and both eyes are injected and very sensitive to light. His left knee and the metatarsophalangeal (MTP) joint of his right great toe are swollen and tender. Which of the following is your diagnosis? A. Gouty arthritis B. Reactive arthritis (Reiter syndrome) C. Infectious arthritis D. Rheumatoid arthritis E. Syphilis

B. The triad of uveitis or conjunctivitis, urethritis, and arthritis is characteristic of reactive arthritis or Reiter syndrome. This poorly understood disease is thought to be caused by immune cross-reaction between antigens in infectious organisms and the host connective tissue. Organisms commonly involved include C trachomatis, which this patient may have contracted when he contracted syphilis but which may not have been treated. The arthritis typically involves large joints and is both progressive and additive. The uveitis can be difficult to treat; however, the dysuria of the urethritis can be transient. Patients with Reiter syndrome are often HLA-B27 positive.

3.2 A 48-year-old woman is noted to have atrial fibrillation with a ventricular heart rate of 140 bpm. She is feeling dizzy and dyspneic with a systolic blood pressure of 75/48 mm Hg. Which of the following is the most appropriate next step? A. Intravenous digoxin B. DC cardioversion C. Vagal maneuvers D. Intravenous diltiazem (Cardizem)

B. This individual has signifi cant symptoms and hypotension caused by the atrial fibrillation and rapid ventricular rate; consequently, DC cardioversion is the treatment of choice.

25.4 A 20-year-old woman with heavy menses MCV Ferritin TIBC RDW A. Increased Decreased Increased Decreased B. Decreased Decreased Increased Increased C. Normal Increased Normal Normal D. Decreased Increased Normal Normal E. Increased Increased Decreased Increased

B. This laboratory finding is diagnostic of iron-defi ciency anemia (microcytic, low ferritin, high TIBC, high RDW).

58.2 A 50-year-old man has been treated for rheumatoid arthritis for many years. He currently is taking corticosteroids for the disease. On examination, he has stigmata of rheumatoid arthritis and some fullness on his left upper abdomen. His platelet count is slightly low at 105 000/mm3. His white blood cell count is 3100/mm3 with neutropenia, and hemoglobin level 9.0 g/dL. Which of the following is the most likely etiology of the thrombocytopenia? A. Steroid induced B. Splenic sequestration C. Autoimmune destruction D. Prior gold therapy

B. This patient with rheumatoid arthritis likely has splenomegaly, also known as Felty syndrome. Splenomegaly from any etiology may cause sequestration of platelets, leading to thrombocytopenia.

60.1 A young woman presents to your office complaining of dizziness. When asked to describe the feeling, she gives a vague story of just feeling like "her head is too big." The feeling is associated with palpitations, sweating, and nervousness, and is almost constant. Her examination, including neurologic evaluation, is completely normal. Which of the following is the best next step? A. Magnetic resonance imaging (MRI) brain scan. B. Obtaining a thorough psychosocial history. C. Dix-Hallpike maneuver. D. Prescribe meclizine. E. Referral to neurology department.

B. This young woman is not describing vertigo. The word "dizzy" can mean several different things, so it is extremely important when obtaining the history to have the patient describe, as best he or she can, what is meant by "dizzy." Patients with vertigo often use descriptors indicating movement, such as "the room is moving around me" or "I'm on a roller coaster." Feelings of dysequilibrium, or "out-of-body" experiences such as this young woman describes, are not typical of vertigo and indicate another problem. It would be important to know what the symptoms are associated with; for example, is there increased stress in her job or intimate relationship? Is this panic disorder or anxiety disorder?

CML pathology

BCR-ABL fusion gene found in the Philadelphia chromosome t(9;22) produces a deregulated tyrosine kinase that is implicated in the pathogenesis of CML, and is the target of therapy.

diagnosis of BPPV and treatment

Benign paroxysmal positional vertigo is the most common cause of vertigo and can be diagnosed by the history of intermittent positional symptoms, absence of other otologic or neurologic findings, and a positive Dix-Hallpike test. Benign positional vertigo can be treated with maneuvers to reposition the abnormal otolith from the semicircular canal or by anticholinergic medications such as meclizine.

46.3 Which of the following patients is the best candidate for lifestyle modification alone rather than lipid-lowering medications? A. A 60-year-old diabetic male smoker with a recent myocardial infarction: cholesterol 201 mg/dL, HDL 47 mg/dL, and LDL 138 mg/dL B. A 62-year-old diabetic man: cholesterol 210 mg/dL, HDL 27 mg/dL, and LDL 146 mg/dL C. A 57-year-old asymptomatic woman: cholesterol 235 mg/dL, HDL 92 mg/dL, and LDL 103 mg/dL D. A 39-year-old man with nephrotic syndrome: cholesterol 285 mg/dL, HDL 48 mg/dL, LDL 195 mg/dL

C. Patient A is at highest risk for future events because he has established CHD and diabetes, he smokes, and he recently had a myocardial infarction. His goal LDL is < 70 mg/dL. Patient B has diabetes, a CHD equivalent. Besides lifestyle modifications, he should start drug therapy to lower his LDL and raise his HDL. Patient C has very high HDL, which is protective, and probably contributes to her elevated total cholesterol. Patient D has nephrotic syndrome causing hyperlipidemia, which may be treated by reduction of proteinuria using angiotensin-converting enzyme (ACE) inhibitors but often requires drug therapy such as statins.

60.3 A 65-year-old woman with a history of benign positional vertigo returns to your office for follow-up. Although manageable, the symptoms of vertigo continue to recur periodically. Between episodes she generally feels normal but occasionally somewhat "off-balance." Today, her neurologic examination is completely normal, except that the thresholds of both air and bone conduction of a vibrating 256-Hz tuning fork are elevated on the left side. Which of the following is the most likely diagnosis? A. Intermittent benign positional vertigo B. Otosclerosis C. Acoustic neuroma D. Acute basilar artery infarct E. Panic disorder

C. Acoustic neuromas are slow-growing tumors of the eighth cranial nerve. Because of the slow growth of the tumor, the neurologic system often is able to accommodate, so patients may have only subtle symptoms that at first may be confused with benign positional vertigo. The keys in this patient's history are the persistent low-grade feelings of dysequilibrium and the finding of probable sensorineural hearing loss on the left side. This finding indicates a possible problem with the eighth nerve, and an MRI would best delineate the anatomy.

22.5 A 36-year-old woman was seen by her physician due to pain in her hands, wrists, and knees. She is diagnosed with rheumatoid arthritis. Which of the following treatments will reduce joint inflammation and slow progression of the disease? A. NSAIDs B. Joint aspiration C. Methotrexate D. Systemic corticosteroids

C. Although NSAIDs and corticosteroids may help to relieve symptoms, they typically do not alter the disease course significantly. Disease-modifying mediations include methotrexate, hydroxychloroquine, sulfasalazine, oral and parenteral gold, and penicillamine. If Pt cannot take methotrexate, Leflunomide could be used as a substitute *Combination of MTX, HCQ and SSZ may be beneficial. Of these agents, methotrexate is thought to be the first line.

34.2 Which of the following findings are you most likely to encounter in an 80-year-old woman with severe kyphoscoliosis? A. Enlarged overall lung volume (TLC) B. Decreased FEV1/FVC C. Decreased vital capacity (VC) D. Increased vital capacity (VC) E. ABG with pH 7.48 and PaCO2 of 32 mm Hg

C. Chest wall deformities can lead to chronic hypoventilation with elevated PaCO2 levels, as well as with recurrent pulmonary infection. The pattern on pulmonary function testing is usually that of a restrictive pattern, with decreased total lung volumes and vital capacity, but with normal FEV1/FVC.

49.1 A 78-year-old woman is diagnosed with early Alzheimer disease. Which of the following agents is most likely to help with the cognitive function? A. Haloperidol B. Estrogen replacement therapy C. Donepezil D. High-dose vitamin B12 injections

C. Cholinesterase inhibitors help with the cognitive function in Alzheimer disease and may slow the progression somewhat.

25.6 A 50-year-old man with severe rheumatoid arthritis MCV Ferritin TIBC RDW A. Increased Decreased Increased Decreased B. Decreased Decreased Increased Increased C. Normal Increased Normal Normal D. Decreased Increased Normal Normal E. Increased Increased Decreased Increased

C. Chronic disease generally leads to a normocytic anemia with elevated ferritin level (acute-phase reactant); although a microcytic anemia can also be seen, a normocytic anemia is more common Expect BM Iron to be abundant and TIBC to be nml to decreased (vs. increased in iron deficiency). *Hepcidin (produced by liver) internalizes ferroportin - which allows for release of iron...and results in decreased transport of iron from intestinal cells into circulation).

21.3 A 30-year-old man is noted to have an acutely swollen and red knee. Joint aspirate reveals numerous leukocytes and polymorphonuclear leukocytes, but no organisms on Gram stain. Analysis shows few negatively birefringent crystals. Which of the following is the best initial treatment? A. Oral corticosteroids B. Intraarticular corticosteroids C. Intravenous antibiotic therapy D. Oral colchicine

C. Corticosteroids should not be used until infection is ruled out. The inflammatory arthritis as shown by Gram stain of the joint aspirate is suspicious for infection, even with no organisms seen on Gram stain. Also, the presence of a few crystals does not eliminate an infection.

59.2 Which of the following statements regarding CML is true? A. Peripheral smear shows elevated WBC count with mature and immature granulocytes, toxic granulation, and high LAP score. B. Usually presents initially with splenomegaly, anemia, and thrombocytopenia. C. Chromosomal translocations, most often t(9;22), are found in 90%-95% of patients. D. Is an indolent disease, and should be monitored without treatment until patients enter accelerated or blast phase.

C. Definitive diagnosis of CML is established by demonstrating the presence of the Philadelphia chromosome or the underlying t(9;22) translocation, the BCR-ABL1 fusion gene or mRNA fusion product, which is found in nearly all patients. Toxic granulation and high LAP score are features of leukemoid reaction. Splenomegaly is common in CML, but significant cytopenias are not seen. Before imatinib and other TKIs, median survival in CML was 4 years with progression to blast (acute leukemic) phase and death. Imatinib or other TKIs are indicated as initial treatment for patients in chronic phase, with the goals of achieving remission and preventing progression of disease.

37.2 A 39-year-old man is noted to have a deep venous thrombosis without any known risk factors. He notes that his brother also developed a pulmonary embolism at age 45 years, and his mother developed a "clot in the leg" when she was in her thirties. Which of the following is the most likely inherited disorder in this patient? A. Protein S deficiency B. Antithrombin III deficiency C. Factor V Leiden mutation D. Antiphospholipid antibody syndrome E. Familial malignancy syndrome

C. Factor V Leiden mutation is the most common hereditary thrombophilia.

44.1 A 44-year-old woman is noted to be nervous and has heat intolerance. Her thyroid gland is diffusely enlarged, nontender, with an audible bruit. Her TSH level is very low. Which of the following is the most likely etiology? A. Lymphocytic thyroiditis B. Hashimoto thyroiditis C. Graves disease D. Multinodular toxic goiter

C. Graves disease is the most common cause of hyperthyroidism in the United States. It often includes the thyroid gland features described, as well as the distinctive eye findings.

39.1 A 65-year-old cigarette smoker with a history of hypertension and mild congestive heart failure presents to the emergency room with worsening cough, fever, and dyspnea at rest. The illness began 1 week ago with fever, muscle aches, abdominal pain, and diarrhea, with nonproductive cough developing later that week and rapidly becoming worse. Therapy for which of the following atypical organisms must be considered in this case? A. Chlamydia pneumoniae B. Mycoplasma pneumoniae C. Legionella pneumophila D. Coccidiomycosis E. Aspergillus fumigatus

C. Legionella typically presents with myalgias, abdominal pain, diarrhea, and severe pneumonia.

35.1 A patient with known asthma undergoing therapy with inhaled corticosteroid and intermittent (short-acting) β2-agonist presents with complaints of nocturnal awakenings secondary to cough and occasional wheezing. This episode occurs three to four times per week. Pulmonary function tests in the past have shown mild obstructive lung disease. Which of the following is the best next step? A. Oral steroids B. Leukotriene inhibitors C. Long-acting β2-agonists D. Theophylline E. Antireflux therapy

C. Long-acting β2-agonists are helpful in this situation. The asthma would be classified as moderate persistent, and the recommended treatment is longacting β2-agonists, such as salmeterol, which are particularly helpful with nocturnal symptoms

58.3 A 30-year-old woman with ITP has been taking maximum corticosteroid doses and still has a platelet count of 20 000/mm3 and frequent bleeding episodes. Which of the following should she receive before her splenectomy? A. Washed leukocyte transfusion B. Intravenous interferon therapy C. Pneumococcal vaccine D. Bone marrow radiotherapy

C. Patients who undergo splenectomy are at risk for infections of encapsulated organisms such as Streptococcus pneumoniae and thus should receive the pneumococcal vaccine. It usually is given 2 weeks prior to splenectomy so that the spleen can help in forming a better immune response.

13.4 A 56-year-old woman who presented with complaints of pruritus and fatigue has elevated alkaline phosphatase. Select the cause that is probably responsible for the patient's presentation. A. Wilson disease B. Hematochromatosis C. Primary biliary cirrhosis D. Sclerosing cholangitis E. Autoimmune hepatitis F. Alcohol-induced hepatitis G. Viral hepatitis

C. Primary biliary cirrhosis is thought to be an autoimmune disease leading to destruction of small- to medium-size bile ducts. Most patients are women between the ages of 35 and 60 years, who usually present with symptoms of pruritus and fatigue. Alkaline phosphatase elevated two to five times above the baseline should raise suspicion; diagnosis is confirmed with antimitochondrial Ab.

44.3 A 58-year-old woman is noted to have Graves disease and has a small goiter. Which of the following is the best therapy? A. Long-term propranolol B. Lifelong oral propylthiouracil (PTU) C. Radioactive iodine ablation D. Surgical thyroidectomy

C. Radioactive iodine is the defi nitive treatment for Graves disease. Surgery is indicated for obstructive symptoms or for women during pregnancy.

42.3 A 75-year-old woman with diabetes for approximately 20 years, diabetic retinopathy, and diabetic nephropathy with creatinine level 2.2 mg/dL is brought into the clinic by her daughter for follow-up. The patient currently takes a sulfonylurea for her diabetes and an ACE inhibitor for her proteinuria. Her daughter reports that, on three occasions in the past 2 weeks, her mother became sweaty, shaky, and confused, which resolved when she was given some orange juice. Which of the following conditions is most likely to be contributing to these episodes? A. Excess caloric oral intake B. Interaction between the ACE inhibitor and the sulfonylurea agents C. Worsening renal function D. Hyperglycemic amnesia

C. Sulfonylureas have long half-lives and can cause prolonged hypoglycemia in elderly patients as well in those with renal insufficiency. Another method, such as insulin, may be more appropriate in this patient, as well as less-intensive control, aiming for an HbA1C of 8% instead of 7%.

37.1 A 35-year-old woman complains of calf tenderness and acute dyspnea. The arterial blood gas reveals PO2 (partial pressure of oxygen) of 76 mm Hg. Which of the following is the most common physical examination finding of pulmonary embolism? A. Wheezing B. Increased pulmonary component of the second heart sound C. Tachypnea D. Calf swelling E. Pulmonary rales

C. Tachypnea is the most common physical sign associated with pulmonary embolus.

9.1 A 30-year-old woman is noted to have blood pressures in the 160/100 mm Hg range. She also has increased obesity, especially around her abdomen, with striae. She has been bruising very easily and has hirsutism. Which of the following is the most likely diagnosis? A. Hyperthyroidism B. Coarctation of the aorta C. Cushing syndrome D. Pheochromocytoma

C. The central obesity, abdominal striae, hirsutism, and easy bruisability are consistent with Cushing syndrome, a disease of adrenal steroid overproduction. Obtain fasting glucose/A1C

35.4 An obese 50-year-old man with a history of asthma returns with complaints of occasional dyspepsia and nocturnal cough. He wakes up in the morning with a sour taste in his mouth. His current medications include inhaled corticosteroid and a short-acting β2-agonist. Which of the following should be your next step? A. 24-Hour esophageal pH monitoring B. Chest radiograph C. Initiation of omeprazole D. Short course of oral corticosteroids E. Initiation of allergy desensitization

C. The dyspepsia and the sour taste suggest GERD. Aside from acid suppression, other recommendations include dietary modifications and weight reduction. **Twenty-four-hour esophageal pH monitoring is indicated only if the medication does not help.

20.2 A 19-year-old man was seen at the university student health clinic a week ago complaining of pharyngitis, and now returns because he has noted discoloration of his urine. He is noted to have elevated blood pressure (178/110 mm Hg), and urinalysis reveals RBC casts, dysmorphic RBCs, and 1+ proteinuria. Which of the following is the most likely diagnosis? A. Systemic lupus erythematosus (SLE) B. Amyloidosis C. Poststreptococcal glomerulonephritis D. HIV nephropathy E. Diabetic nephropathy

C. The patient has hypertension, and urinary sediment consistent with a nephritic rather than nephrotic syndrome (RBC casts, mild degree of proteinuria). Given his recent episode of pharyngitis, the most likely cause would be postinfectious, probably due to streptococcal infection. SLE can produce a variety of renal diseases, including both nephritic and nephrotic manifestions, but it would be unlikely in a male patient, especially without other clinical manifestations of lupus such as arthritis. Amyloidosis, diabetes, and HIV all cause renal disease, but usually produce the nephrotic syndrome (heavy proteinuria >3 g/d, edema, hypoalbuminemia).

54.2 Which of the following is the best treatment for a 39-year-old woman with fever of 103°F, nausea, flank pain, and more than 105 CFU/mL of E coli in a urine culture? A. Oral trimethoprim-sulfamethoxazole for 3 days B. Single-dose ciprofloxacin C. Intravenous and then oral gatifloxacin for 14 days D. Oral ampicillin for 21-28 days

C. The patient in this scenario has symptoms of upper tract infection, for example, pyelonephritis, and is moderately ill with nausea. She will need a 14-day course of treatment and may not be able to take oral antibiotics initially, so hospitalization and treatment with intravenous antibiotics likely will be necessary. Single-dose and 3-day regimens are useful only for acute uncomplicated cystitis in women. E coli is frequently resistant to ampicillin.

54.3 A 57-year-old man is noted to have a blood pressure 68/50 mm Hg, heart rate 140 bpm, elevated jugular venous pressure, inspiratory crackles on examination, and cold clammy extremities. Which of the following is the most likely etiology? A. Septic shock B. Adrenal crisis C. Cardiogenic shock D. Hypovolemic shock

C. The patient is hypotensive with signs of left- and right-heart failure, that is, probably cardiogenic shock. Septic shock and adrenal crisis both are forms of distributive shock that would produce warm extremities. Hypovolemic shock should have flat neck veins and no pulmonary edema.

47.4 A 62-year-old man who works at an automobile assembly line has noticed that he feels pain, fatigue, and numbness in his right arm while working for the last several months. This morning at work, he noticed vertigo, then lightheadedness, then lost consciousness for a few seconds. The blood pressure in his right arm is 30 mm Hg lower than that in his left arm. What is the most likely diagnosis? A. Left middle cerebral artery stroke B. Lacunar infarction involving right internal capsule C. Stenosis of right subclavian artery due to atherosclerosis D. Multiple sclerosis

C. The patient likely has subclavian steal: phenomenon of flow reversal in the vertebral artery ipsilateral to a hemodynamically significant stenosis of the subclavian artery. The neurologic symptoms can be caused by vertebrobasilar ischemia.

41.3 A 25-year-old man presents with 3 days of scleral icterus but has been otherwise feeling well. His laboratory results are AST 45 U/L, ALT 48 U/L, alkaline phosphatase 100 U/L, total bilirubin 3.2 mg/dL, direct bilirubin 0.2 mg/dL, and albumin 3.5 g/dL. Complete blood count and lactate dehydrogenase (LDH) are normal. A. Hemolysis B. Alcoholic hepatitis C. Gilbert disease D. Pancreatic cancer E. Gallstones F. Primary sclerosing cholangitis

C. The patient's laboratory results show an unconjugated hyperbilirubinemia without other abnormality. He is otherwise healthy without symptoms of systemic disease or hemolytic anemia. No treatment is necessary.

46.1 A 35-year-old man with no history of cardiac or other vascular disease asks how often he should have routine cholesterol screening. Which of the following is the best answer? A. Every 3 months B. Annually C. Every 5 years D. Every 7-10 years

C. The recommended interval for cholesterol screening in this population of healthy adults is every 5 years. Cholesterol levels do not change rapidly over a person's lifetime. A rapid change should prompt investigation for an underlying secondary cause.

2.3 A 75-year-old man is noted to have chest pain with exertion and has been passing out recently. On examination he is noted to have a harsh systolic murmur. Which of the following is the best therapy for his condition? A. Coronary artery bypass B. Angioplasty C. Valve replacement D. Carotid endarterectomy

C. The symptoms of aortic stenosis classically progress through angina, syncope, and, finally, congestive heart failure, which has the worse prognosis for survival. This patient's systolic murmur is consistent with aortic stenosis. An evaluation should include echocardiography to confirm the diagnosis, and then aortic valve replacement.

53.2 A 63-year-old man has had annual health maintenance appointments and has followed all the recommendations offered by his physician. The physician counsels him about varicella zoster vaccine. Which of the following is the most accurate statement about this vaccine? A. This vaccine is recommended for patients who are aged 65 and older. B. This vaccine is not recommended if a patient has already developed shingles. C. This vaccine is a live attenuated immunization. D. This vaccine has some cross-reactivity with herpes simplex virus and offers some protection against HSV.

C. The varicella zoster vaccine is a live attenuated vaccine, recommended for individuals aged 60 and above, and has been shown to greatly reduce the incidence of herpes zoster (shingles) and the severity and likelihood of postherpetic neuralgia. It has no efficacy in preventing HSV. New: Shingrix preferred over Zostavax. Shingrix = recombinant (NO LONGER LIVE) for 50 years or older: two doses- 2 to 6 months apart - immunocompetent patients over 50 years and others with chronic disease based on guidelines. No need to screen for past varicella infection.

15.2 A 67-year-old woman has diabetes and mild hypertension. She is noted to have some diabetic retinopathy, and she states that she cannot feel her legs. She has recurrent episodes of lightheadedness when she gets up in the morning. She comes in now because she had fainted this morning. Which of the following is the most likely cause of her syncope? A. Carotid sinus hypersensitivity B. Pulmonary embolism C. Autonomic neuropathy D. Critical aortic stenosis

C. This diabetic patient has evidence of microvascular disease, including peripheral neuropathy, and likely has autonomic dysfunction.

39.2 An 85-year-old nursing home resident with a history of congestive heart failure has dementia such that she requires assistance in all activities of daily life. She has a 3-day history of fever and productive cough. Chest x-ray reveals a right middle lobe consolidation. Which of the following is the most appropriate initial antibiotic choice? A. Oral amoxicillin B. Intravenous linezolid C. Intravenous cefepime D. Oral azithromycin

C. This nursing home resident would be considered to have a nosocomial rather than community-acquired infection, with a higher incidence of gramnegative infection. Her age and comorbid medical conditions place her at high risk, requiring hospitalization for intravenous antibiotics such as a thirdgeneration cephalosporin. outpatient CAP can usually be treated w/ macrolide antibiotics (azithromycin)

24.3 A 70-year-old woman presents with a 4-week history of low back pain, generalized weakness, and a 15-lb weight loss over the last 2 months. Her medical history is unremarkable, and her examination is normal except that she is generally weak. Initial laboratory tests reveal an elevated sedimentation rate, mild anemia, creatinine level 1.8 mg/dL, and calcium level 11.2 mg/dL. Which of the following is the most likely diagnosis? A. Osteoporosis with compression fractures B. Renal failure with osteodystrophy C. Multiple myeloma D. Lumbar strain E. Osteomyelitis

C. This patient has many "red fl ag" symptoms in her presentation: her age, new-onset pain, and history of weight loss. The elevated calcium level and mild renal failure are classic for multiple myeloma. Plain radiographs of the spine and, more likely, of the skull may illustrate the punched out lytic bone lesions often seen in this disease. Bence Jones protein in the urine is also a finding in multiple myeloma.

44.2 Which of the following distinguishes hyperthyroidism from thyroid storm? A. Tachycardia to heart rate 120 bpm B. Weight loss C. Fever and delirium D. Large goiter

C. Thyroid storm is an exaggeration of hyperthyroid features with extreme tachycardia (heart rate >140 bpm), fever, and central nervous system dysfunction, such as confusion or coma. It is a medical emergency with a high mortality.

42.2 A 45-year-old obese Hispanic woman presents for follow-up of her diabetes. She currently takes metformin 1000 mg twice per day, and her fasting morning glucose runs approximately 170 to 200 mg/dL. Her last HbA1C was 7.9. She states that she conscientiously follows her diet and that she walks 30 minutes to 1 hour daily. Which of the following is the best next step in her care? A. Refer to an endocrinologist for an insulin pump. B. Stop metformin, and start on glimepiride. C. Add once-a-day injection of insulin glargine (Lantus). D. Hospitalize her urgently.

C. When patients fail to achieve glycemic goal (A1C <7.0%) using metformin and lifestyle modifications, the next step is to either add a once-daily basal insulin injection (a long-acting insulin such as NPH, glargine, or detemir) or a sulfonylurea to the regimen. Switching from one class of oral agent to another with similar potency would add no benefi t.

7.3 A 44-year-old woman infected with HIV is noted to have a CD4 count of 180 cells/mm3. Which of the following is recommended as a useful prophylactic agent in this patient at this point? A. Fluconazole B. Azithromycin C. Trimethoprim-sulfamethoxazole D. Ganciclovir

C. When the CD4 count falls to less than 200 cells/mm3, trimethoprimsulfamethoxazole (Bactrim) prophylaxis is generally initiated to prevent PCP. toxoplasmosis prophylaxis usually is started when the CD4 count is less than 100 cells/mm3 (TMP/SMX) Prophylaxis against Mycobacterium avium-intracellulare complex usually is started when the CD4 count is less than 50 cells/mm3 (clarithromycin or Azithromycin) There is no prophylaxis for fungi, HSV, CMV

55.3 Which of the following patients is not a good candidate for evaluation for celiac disease, with either endoscopy or serologic testing? A. A 26-year-old woman who experiences with intermittent abdominal bloating but no diarrhea and is found to have osteopenia and vitamin D deficiency. B. A 19-year-old college freshman with bulky, foul-smelling, floating stools and excessive flatulence, who has lost 20 lb unintentionally. C. A thin, 39-year-old man with a family history of celiac disease, who has been adhering to a gluten-free vegetarian diet for the last 3 years, and now complains of gassiness and reflux. D. A 42-year-old man who was found to have iron deficiency anemia, but has no gastrointestinal symptoms, and recently had a negative colonoscopy.

C. While GI symptoms in a patient with a family history of celiac disease are reasonable to investigate, the fact that he has been on a gluten-free diet for a prolonged period greatly diminishes the sensitivity of both endoscopic and serologic testing. Unexplained osteopenia and vitamin D deficiency in a young woman, unexplained iron deficiency anemia in any patient, and the classic presentation with steatorrhea and weight loss should all be investigated.

CLL course and complications

CLL/SLL is an indolent disease characterized by the monoclonal proliferation of mature B-lymphocytes expressing the CD5 antigen, and typically presents as asymptomatic lymphocytosis or painless lymphadenopathy. Complications of CLL include autoimmune hemolytic anemia or thrombocytopenia, recurrent infections due to immune dysfunction, or transformation to a more aggressive large cell lymphoma

exclusion of CNS syphilis

Central nervous system involvement can be excluded only through testing of the cerebrospinal fluid.

Quick way to exclude most (but not all!) serious and uncommon causes of chronic cough

Chest Radiograph would show: -sarcoid

CURB-65 criteria for pneumonia severity approximation (5 point scale)

Confusion Urea>20 g/dL RR > 30 BP < 90 systolic Age > 65 Score 0: can be manated outpatient if score > 2 - consider hospital management (mortality 9%)

onset of ACEi cough

Cough caused by ACE inhibitors can be triggered after the first dose or may occur after months of therapy.

15.1 An 18-year-old woman is brought to the emergency room because she fainted at a rock concert. She apparently recovered spontaneously, did not exhibit any seizure activity, and has no medical history. Her heart rate is 90 bpm and blood pressure 110/70 mm Hg. Neurologic examination is normal. The pregnancy test is negative, and ECG shows normal sinus rhythm. Which of the following is the most appropriate management? A. Admit to hospital for cardiac evaluation. B. Obtain an outpatient echocardiogram. C. Use 24-hour Holter monitor. D. Reassure the patient and discharge home.

D. A young patient without a medical history and with no seizure activity with a history suggestive of emotionally mediated vasovagal syncope has an excellent prognosis. Reassurance, avoidance of provocative stimuli, and plasma volume expansion with fluid and salt are the cornerstones of the management of neurally mediated syncope. Isometric counterpressure maneuvers of the limbs (leg crossing or handgrip and arm tensing) may raise blood pressure and, by maintaining pressure in the autoregulatory zone, avoid or delay the onset of syncope. Randomized controlled trials support this intervention. Fludrocortisone, vasoconstricting agents, and beta-adrenoreceptor antagonists are widely used by experts to treat refractory patients, although there is no consistent evidence from randomized, controlled trials for any pharmacotherapy to treat neurally mediated syncope. Because vasodilation is the dominant pathophysiologic syncopal mechanism in most patients, use of a cardiac pacemaker is rarely beneficial. Possible exceptions are older patients in whom syncope is associated with asystole or severe bradycardia, and patients with prominent cardioinhibition due to carotid sinus syndrome. In these patients, dual-chamber pacing may be helpful.

36.4 A 66-year-old man with known metastatic squamous cell carcinoma of the esophagus is brought to the emergency room for increasing lethargy and confusion. He is clinically dehydrated, his serum calcium level is 14 mg/dL, and his creatinine level is 2.5 mg/dL but 1 month ago was 0.9 mg/dL. Which therapy for his hypercalcemia should be instituted first? A. Intravenous bisphosphonate B. Intravenous furosemide C. Glucocorticoids D. Intravenous normal saline E. Chemotherapy for squamous cell carcinoma

D. Although all of the other therapies listed may be helpful in the treatment of hypercalcemia, given the clinical findings of dehydration and elevated creatinine level with a history of previously normal renal function, volume expansion with normal saline would correct the dehydration and presumed prerenal azotemia, allowing the kidneys to more efficiently excrete calcium. Other therapies can be added if the response to normal saline alone is insufficient.

39.3 A 56-year-old man is brought into the emergency room intoxicated with alcohol. He has repeated bouts of emesis and is found choking. Lung examination reveals some crackles in the right lung base. Which of the following is the most appropriate management? A. Initiate azithromycin. B. Initiate corticosteroid therapy. C. Initiate haloperidol therapy. D. Observation with follow-up chest radiograph.

D. Antibiotic therapy is generally not indicated for aspiration pneumonitis, but patients need to be observed for clinical deterioration.

33.4 A 28-year-old woman is noted to have a nontender ulcer of the vulva. A herpes culture is taken of the ulcer scraping, which is negative, and the RPR titer is negative. Which of the following is the next best step? A. Empiric treatment with doxycycline for Chlamydia trachomatis B. Empiric treatment with acyclovir for herpes simplex virus (HSV) C. Empiric treatment with azithromycin for Haemophilus ducreyi D. Dark-field microscopy E. Biopsy for possible vulvar cancer

D. Approximately one-third of patients who have the primary lesion of the chancre will have negative serology and require either dark-field microscopy or biopsy with special stains to identify the spirochetes. The organism is too thin to be visualized by conventional light microscopy. Empiric treatment with penicillin is reasonable if dark-field microscopy is not available. Genital herpes and chancroid should produce painful genital ulcers, and Chlamydia should cause non-ulcerative cervicitis or urethritis.

20.1 A 49-year-old woman with type 2 diabetes presents to your office for newonset swelling in her legs and face. She has no other medical problems and says that at her last ophthalmologic appointment she was told that the diabetes had started to affect her eyes. She takes glyburide daily for her diabetes. Physical examination is normal except for hard exudates and dot hemorrhages on funduscopic examination, and diminished sensation up to the mid-shin bilaterally. Her blood pressure is normal. Urine analysis shows 2+ protein and 2+ glucose (otherwise negative). Which of the following is the best treatment for this patient? A. Have the patient return in 6 weeks and check a repeat urine analysis at that time. B. Start metoprolol. C. Change the glyburide to glipizide and have the patient return for followup in 6 weeks. D. Start lisinopril. E. Refer the patient to a cardiologist.

D. Beta-blockers are a good first-choice agent for a patient with hypertension and no comorbidities. However, for the patient with diabetes and nephropathy described in the clinical vignette, the benefit of an ACE inhibitor for decreasing proteinuria makes this the best choice for initial treatment. Changing from one sulfonylurea to another is of no benefit because all are equally efficacious. There is no indication for referral to a cardiologist based on the information provided in the vignette

10.3 A 61-year-old man with coronary artery disease complains of progressive orthopnea and pedal edema. He is hospitalized with a blood pressure of 190/105 mm Hg. Cardiac enzyme levels and ECG are normal. Intravenous furosemide has been administered. Which of the following is the best next step? A. Prescribe a beta-blocker to decrease myocardial oxygen demands. B. Start intravenous dopamine. C. Observe. D. Start an ACE inhibitor.

D. Elevated blood pressures may exacerbate congestive heart failure and must be treated. Generally, beta-blockers are avoided when patients are volume overloaded because beta-blockers decrease myocardial contractility.!!! ACE inhibition reduces afterload, and oral nitrates or IV nitroglycerine reduce preload, and are used to treat acute heart failure.

51.4 You see a 70-year-old woman in your office for a routine checkup, and you order a DEXA scan for bone mineral density screening. The T score returns as −2.5 SD in the spine and −2.6 in the hip. Which of the following statements is most accurate? A. This patient has osteopenia. B. Estrogen replacement therapy should be started with an anticipated rebuilding of bone mass to near-normal within 1 year. C. Swimming will help build bone mass. D. Bisphosphonates would reduce the risk of hip fracture by 30%-50%.

D. Estrogen primarily inhibits loss of bone mass, although it can help to build a modest amount of bone mass, but also may be associated with increased thrombotic and cardiovascular risk. Weight-bearing exercise, and not swimming, is important in preventing osteoporosis. *Bisphosphonates decrease the incidence of hip fractures by 30% to 50%.

10.4 A 58-year-old woman with aphasia and right-arm weakness of 8 hours' duration is seen in the ER. CT scan shows no intracranial hemorrhage. Her blood pressure is 162/98 mm Hg. Which of the following is the best next step? A. Normalize the blood pressure with beta-blockade. B. Admit to ICU with sodium nitroprusside. C. Normalize the blood pressure with an ACE inhibitor. D. Observe the blood pressure.

D. In general, blood pressure should not be acutely decreased in an individual suspected of having a stroke because of the concern for cerebral hypoperfusion and worsening brain ischemia. If thrombolytic therapy is considered, blood pressure should be controlled to <185/100 mm Hg, but this patient's symptom duration precludes that consideration.

46.2 A 38-year-old man presents to your clinic following a health fair screening of his cholesterol level because he was told that it is high. He watches his diet, plays tennis, exercises 3 to 5 times per week, and appears to be in good physical condition. He is a nonsmoker and has no family history of cardiovascular disease. His profile is total cholesterol 202 mg/dL, HDL 45 mg/dL, LDL 128 mg/dL, and triglycerides 145 mg/dL. Following a review of this patient's profile, which of the following would you recommend? A. Administer gemfibrozil. B. Administer HMG-CoA reductase inhibitor. C. Administer low-dose niacin and slowly increase to achieve 3 g daily. D. Suggest he continue his current diet and exercise program.

D. In this scenario, this 38-year-old man's only risk factor for CHD is male sex; thus, his 10-year risk is less than 10%. His total cholesterol is barely in the borderline high category, fairly near the desirable level, his LDL is less than 130 mg/dL, and his HDL is acceptable. *Start on statin if LDL>160

60.4 Which of the following is the best next step for the patient with presumed acoustic neuroma A. Prescription for a selective serotonin reuptake inhibitor B. Referral for a hearing aid C. Lumbar puncture and serology for syphilis D. Referral for an MRI E. Reassurance

D. MRI is the diagnostic test of choice. See answer to Question 60.3.

11.2 What postexposure prophylaxis should the student described in (needle stick w/ pt known to have Hep B/C) receive? A. Hepatitis B immunoglobulin (HBIg) B. Oral lamivudine C. Intravenous immunoglobulin (IVIG) D. Reassurance

D. No postexposure prophylaxis is definitively indicated. The student has detectable protective antibody levels against the hepatitis B virus, and if the levels are judged to be adequate, the student is protected against infection. Oral lamivudine is a treatment for chronic hepatitis B infection and is part of an antiretroviral prophylaxis if the patient was HIV positive. There is no effective prophylaxis for hepatitis C exposure.

55.1 Which of the following features is not consistent with the diagnosis of irritable bowel syndrome? A. Abdominal pain relieved with defecation B. Sensation of incomplete evacuation C. Passage of mucus D. Nocturnal awakening with pain or diarrhea E. Normal bowel habits alternating with either diarrhea or constipation

D. Nocturnal diarrhea is not typically associated with IBS, and should prompt further investigation, for example, with imaging or colonoscopy. The other symptoms listed are included in commonly used diagnostic criteria for IBS. It should be remembered that IBS is essentially a diagnosis of exclusion, and is established when patients have typical symptoms, but other conditions with similar clinical presentations have been excluded in a cost-effective manner.

35.2 Which of the following is most accurate? A. Cough caused by captopril may resolve with switching to enalapril. B. Initial treatment of a chronic cough should include codeine or a similar opiate derivative to suppress the cough. C. Cough caused by reflux can be effectively ruled out by a negative history of heartburn or dyspepsia. D. More than one condition often is responsible for causing a chronic cough in a given patient.

D. Often more than one condition is responsible for causing a chronic cough in a given patient. Cough from ACE inhibitors is class dependent, and change to another class of antihypertensives is more appropriate. The etiology of chronic cough should be determined prior to suppression of the cough because treatment of the underlying condition is the most effective approach. The GERD may present with the sole manifestation of cough, or it may present "silently."

20.4 A 58-year-old man with type 2 diabetes is normotensive, has no known heart disease, and has a baseline creatinine of 1.8 mg/dL. His fasting lipid profile shows triglycerides 205 mg/dL, total cholesterol 220 mg/dL, HDL 35 mg/dL, and LDL 148 mg/dL. What is the most appropriate treatment? A. Niacin B. Low-protein diet C. Gemfibrozil D. Simvastatin

D. Patients with diabetes are considered at high risk for the development of coronary artery disease, and should be treated with lipid-lowering agents such as statins to achieve an LDL less than 100 mg/dL.

48.4 A 57-year-old woman with congestive heart failure (CHF) has a positive cardiac stress test. Cardiac catheterization is required to evaluate for coronary bypass grafting. She states that she has an allergy to iodine. Which of the following is the best next step? A. Desensitization with increasing doses of oral iodine B. Infusion of diphenhydramine during the procedure C. Cancel the procedure and proceed to surgery D. Diphenhydramine and corticosteroids the night before the procedure

D. Pretreatment with diphenhydramine, H2 blockers, and corticosteroids beginning 12 hours before the procedure greatly decreases the reaction to contrast dye.

13.3 A 35-year-old man presents to your clinic with ulcerative colitis. Choose the cause that is probably responsible for the patient's presentation. A. Wilson disease B. Hematochromatosis C. Primary biliary cirrhosis D. Sclerosing cholangitis E. Autoimmune hepatitis F. Alcohol-induced hepatitis G. Viral hepatitis

D. Sclerosing cholangitis is an autoimmune destruction of both the intrahepatic and extrahepatic bile ducts and often is associated with inflammatory bowel disease, most commonly ulcerative colitis. Patients present with jaundice or symptoms of biliary obstruction; cholangiography reveals the characteristic beading of the bile ducts.

21.2 Which of the following diagnostic tests is most likely to give the diagnosis for the case in (10 yo w/ presumed disseminated gonococcal infection - including arthritis) A. Crystal analysis of the joint fluid B. Culture of joint fluid C. Blood culture D. Cervical culture

D. Synovial fl uid cultures usually are sterile in gonococcal arthritis (in fact, the arthritis is more likely caused by immune complex deposition than by actual joint infection), and blood cultures are positive less than 50% of the time. Diagnosis is more often made by finding gonococcal infection in a more typical site, such as urethra, cervix, or pharynx.

25.5 A 34-year-old man of Mediterranean descent with a family history of anemia MCV Ferritin TIBC RDW A. Increased Decreased Increased Decreased B. Decreased Decreased Increased Increased C. Normal Increased Normal Normal D. Decreased Increased Normal Normal E. Increased Increased Decreased Increased

D. Thalassemia usually leads to a microcytic anemia with uniform red cell size (normal RDW) and excess iron stores.

49.3 A 55-year-old man is noted by his family members to be forgetful and become disoriented. He has difficulty making it to the bathroom in time and complains of feeling as though "he is walking like he was drunk." Which of the following therapies is most likely to improve his condition? A. Intravenous penicillin for 21 days B. Rivastigmine C. Treatment with fluoxetine for 9 to 12 months D. Ventriculoperitoneal shunt E. Enrollment into alcoholic anonymous

D. The classic triad for normal pressure hydrocephalus is dementia, incontinence, and gait disturbance; one treatment is shunting the cerebrospinal fl uid.

59.1 A 25-year-old man presents with a 2-week history of low-grade fever, slight cough, malaise, and myalgias, and is noted on physical examination to have enlarged posterior cervical lymph nodes and significant splenomegaly. His CBC shows a lymphocytosis with ALC 10 000/μL, normal hemoglobin level, and normal platelet count. The peripheral smear shows large atypical lymphocytes. What is the most likely diagnosis? A. ALL B. CLL C. Acute HIV infection D. EBV infection E. Pertussis

D. The clinical presentation of fever, malaise, adenopathy, and splenomegaly is consistent with infectious mononucleosis, which is most often associated with EBV, but can also be due to CMV or other viral infections. The absence of cytopenias makes ALL unlikely. The lymphocytosis in CLL and pertussis consists of mature small lymphocytes. Acute HIV infection can present similarly to mononucleosis, but does not typically cause massive splenomegaly.

56.5 Unilateral nontender bony enlargement of the first DIP and activity-related right hip pain in a 68-year-old woman A. Gonococcal arthritis B. Gout C. Pseudogout D. Osteoarthritis E. Rheumatoid arthritis F. Systemic lupus erythematosus

D. The location and asymmetry of joint involvement, lack of infl ammatory signs, and worsening with exertion all are characteristic of OA.

7.4 A 36-year-old woman with HIV is admitted with new-onset seizures. The CT scan of the head reveals multiple ring-enhancing lesions of the brain. Which of the following is the best therapy for the likely condition? A. Rifampin, isoniazid, ethambutol B. Ganciclovir C. Penicillin D. Sulfadiazine with pyrimethamine

D. The most common cause of a mass lesion of the brain in an HIV patient is toxoplasmosis, which is treated with sulfadiazine with pyrimethamine

58.4 A 65-year-old man who has a prosthetic heart valve is hospitalized for a knee replacement surgery, and placed on IV heparin for anticoagulation before the procedure. He drinks one glass of wine each weekend and has been diagnosed with osteoarthritis for which he takes acetaminophen. His platelet count was normal, but now is 32 000/mm3. Which of the following is the most likely cause of the thrombocytopenia? A. Prosthetic heart valve B. Alcohol intake C. Acetaminophen D. Heparin

D. The patient likely has heparin-induced thrombocytopenia, which may be confi rmed by assay for HIT antibodies. Treatment consists of stopping the heparin.

7.2 Which of the following is the most likely organism to cause a lobar pneumonia in a patient with AIDS? A. Pneumocystis jirovecii B. Mycobacterium tuberculosis C. Histoplasmosis capsulatum D. Streptococcus pneumoniae

D. The same organisms that cause community-acquired pneumonia in immunocompetent individuals are causative in HIV patients. Additionally, HIV patients may be more susceptible to encapsulated organisms such as S pneumoniae and H influenzae.

50.1 A 42-year-old man with polycystic kidney disease who complained of a sudden onset of severe headache and then lost consciousness A. Migraine headache B. Tension headache C. Cluster headache D. Subarachnoid hemorrhage E. Meningitis

D. The sudden onset of severe headache with diminution in level of consciousness is classic for subarachnoid hemorrhage. This patient likely had rupture of a cerebral artery aneurysm, which is associated with polycystic kidney disease.

3.4 A 59-year-old woman has been placed on warfarin (Coumadin) after being found to have had chronic atrial fibrillation. She is noted to have an INR of 5.8, is asymptomatic, and has no overt bleeding. Which of the following is the best management for this patient? A. Transfuse with erythrocytes. B. Give vitamin K. C. Give fresh-frozen plasma. D. Hold warfarin.

D. The target INR with warfarin is 2 to 3; thus, 5.8 is markedly elevated. However, because she has no overt bleeding and is asymptomatic, holding the warfarin until the INR reaches the acceptable range is a reasonable approach (as earlier mentioned, if INR is 6-9 and no overt bleeding, hold Coumadin).

58.1 A 28-year-old woman complains of excessive bleeding from her gums and has petechiae. Her CBC shows a platelet count of 22,000/mm3 with a hemoglobin of 8.9 g/dL and a WBC count of 87,000/mm3. Which of the following is the most likely etiology of her low platelet count? A. Immune thrombocytopenia purpura B. Systemic lupus erythematosus C. Drug-induced thrombocytopenia D. Acute leukemia

D. The thrombocytopenia is seen with other hematologic abnormalities, the most abnormal of which is a markedly elevated WBC count, suggesting acute leukemia

24.4 A 45-year-old man complains of decreased sensation in his buttocks and inability to achieve an erection. On examination he has decreased anal sphincter tone and decreased ankle reflexes bilaterally. Which of the following is the next best step in management? A. Bed rest and follow-up in 4 to 6 weeks B. Plain film x-ray of lumbosacral spine C. Sedimentation rate and complete blood count D. Immediate referral for surgical decompression

D. This individual has cauda equine syndrome and requires immediate surgical decompression to avoid long-term nerve denervation and incontinence/lower extremity weakness. The decreased anal sphincter tone and decreased ankle reflexes indicate a peripheral neuropathy. Bed rest with follow-up is indicated when no "red flag" symptoms and signs are present. The plain film x-ray is often normal in patients with cauda equina syndrome

3.5 A 45-year-old woman is noted to have dizziness, pounding of the chest, and fatigue of 3 hours' duration. On examination, she is noted to have a blood pressure (BP) of 110/70 mm Hg and heart rate of 180 bpm. She is noted on ECG to have atrial fibrillation, and a prior baseline ECG showed delta waves. The ER physician counsels the patient regarding cardioversion, but the patient declines. Which of the following is the best therapy for her condition? A. Digoxin B. Angiotensin-converting enzyme (ACE) inhibitor C. Calcium channel blocker D. Procainamide

D. This patient has atrial fibrillation but with WPW as indicated by the delta wave. In this setting, the typical agents used to treat atrial fibrillation that slow the AV node are contraindicated since the conduction through the accessory pathway could actually accelerate. DC cardioversion is an option; however in a hemodynamically stable patient, procainamide may be used since it will slow propagation through the accessory pathway. Because this patient declines cardioversion, procainamide is the best choice.

34.3 A 56-year-old woman admits to a 60-pack-year smoking history. She complains of fatigue and dyspnea with minimal exertion, and a cough that is productive each morning. Which of the following is the most likely finding in this patient? A. Normal diffusing capacity of lung for carbon monoxide (DLCO) B. Decreased residual volume C. Normal to slightly increased forced expiratory volume in first second (FEV1) D. Decreased forced expiratory volume in first second/forced vital capacity (FEV1/FVC) E. Decreased forced vital capacity (FVC)

D. This patient likely has COPD, based on the smoking history and symptoms. A decrease in the forced expiratory volume in first second/forced vital capacity ratio is the hallmark of airflow obstruction. The FEV1 is decreased in obstructive, as well as in restrictive, lung disease. The diffusing capacity is typically deceased in COPD as well as intrinsic restrictive lung disease. The DLCO indicates the adequacy of the alveolar-capillary membrane; the residual volume is the volume of air remaining in the lungs after a maximal expiratory effort and is usually increased in COPD due to air trapping.

33.2 As part of normal screening during pregnancy, a 28-year-old G2P1 has a positive RPR with a titer of 1:64 and a positive MHA-TP. She is allergic to penicillin, which causes shortness of breath and "swelling of her tongue." Which of the following treatments do you offer? A. Erythromycin estolate. B. Doxycycline. C. Tetracycline. D. Penicillin after desensitization. E. Vancomycin. F. Wait until delivery of the baby before treatment.

D. This patient should be desensitized and treated with penicillin, especially because she is pregnant and may pass the disease to her child. Following treatment, her titers should be closely followed and should show at least a fourfold decrease. Treatment of the child after delivery with intravascular (IV) penicillin should be considered.

C. The patient is hypotensive with signs of left- and right-heart failure, that is, probably cardiogenic shock. Septic shock and adrenal crisis both are forms of distributive shock that would produce warm extremities. Hypovolemic shock should have flat neck veins and no pulmonary edema.

D. When septic shock is refractory to volume resuscitation, then vasopressors such as dopamine or norepinephrine are generally the next step. Corticosteroids can be administered empirically if hypotension is refractory to pressors. Intravenous morphine might lower his blood pressure further. FFP is used when the patient shows evidence of coagulopathy such as disseminated intravascular coagulation.

68-year-old female is screened for hypothyroidism and is found to have a TSH 9 (nl 0.4 - 4.5) and freeT4 0.9 (nl 0.8 - 2.7). She has no symptom of hypothyroidism. Diagnosis and followup Goal TSH

Dx: Subclinical hypothyroidism (decreased thyroid gland function; requires increased TSH) once TSH >10, repeat 3 months later...if still elevated, consider starting with T2 and monitor TSH every 4-6 wks Goal TSH <2

56.6 A 72-year-old man complains of painful joints in his hips and knees, which you have diagnosed as osteoarthritis. Which of the following is the best agent to prescribe for this patient? A. Naproxen sodium B. Celecoxib C. Oral prednisone D. Intra-articular prednisone E. Acetaminophen

E. Acetaminophen is the first agent of choice in the treatment of early osteoarthritis. Pharmacologic therapy -acetaminophen -NSAIDs: may be more effective than APAP but have increased risk (GI, CV)...topical NSAIDs may be safer -analgesics: tramadol and opioid analgesics: problems with tolerance and potential addiction (although rare) -topical agents: capsaicin, trolamine salicylate and lidocaine -injection therapy -corticosteroids: effective at least in the short term -visocosupplementation: possibly effective (Jury is still out)

4.2 Which of the following is the most accurate statement regarding H pylori infection? A. It is more common in developed than underdeveloped countries. B. It is associated with the development of colon cancer. C. It is believed to be the cause of nonulcer dyspepsia. D. The route of transmission is believed to be sexually transmitted. E. It is believed to be a common cause of both duodenal and gastric ulcers.

E. Although H pylori is clearly linked to gastric and duodenal ulcers and probably to gastric carcinoma and lymphoma, whether it is more common in patients with nonulcer dyspepsia and whether treatment in those patients reduces symptoms are unclear. It is more common in underdeveloped or developing countries.

37.3 A 54-year-old woman is noted to have cervical cancer and presents with significant vaginal bleeding with a hemoglobin level of 7 g/dL. Her left leg is swollen, which on Doppler investigation reveals a deep venous thrombosis. Which of the following is the best treatment for the thrombus? A. Intravenous unfractionated heparin B. Fractionated subcutaneous heparin C. Subcutaneous unfractionated heparin D. Oral warfarin (Coumadin) E. Vena cava filter

E. Cervical cancer with significant vaginal bleeding is a relative contraindication for anticoagulation. Thus, a vena cava filter is the most appropriate choice in this patient.

34.4 Which of the following therapies is most likely to provide the greatest benefit to a patient with chronic stable emphysema and a resting oxygen saturation of 86%? A. Inhaled tiotropium daily B. Inhaled albuterol as needed C. Oral prednisone daily D. Supplemental oxygen used at night E. Supplemental oxygen used continuously

E. For patients with chronic hypoxemia, supplemental oxygen has a significant impact on mortality, with a greater benefit with continuous usage, rather than intermittent or nocturnal-only usage. Bronchodilators such as tiotropium and albuterol improve symptoms and improve FEV1, but offer no mortality benefit. Chronic use of oral corticosteroids should be avoided because of unfavorable side effects such as osteoporosis, glucose intolerance, and gastrointestinal (GI) side effects.

13.1 A 15-year-old adolescent female has elevated liver enzymes and a positive antinuclear antibody (ANA). Choose the one cause (A-G) that is probably responsible for the patient's presentation. A. Wilson disease B. Hematochromatosis C. Primary biliary cirrhosis D. Sclerosing cholangitis E. Autoimmune hepatitis F. Alcohol-induced hepatitis G. Viral hepatitis

E. Idiopathic or autoimmune hepatitis is a less-well-understood cause of hepatitis that seems to be caused by autoimmune cell-mediated damage to hepatocytes. subgroup of these patients includes young women with positive ANAs and hypergammaglobulinmia who may have other symptoms and signs of systemic lupus erythematosus

22.1 A 72-year-old man develops severe pain and swelling in both knees, shortly after undergoing an abdominal hernia repair surgery. Physical examination shows warmth and swelling of both knees with large effusions. Arthrocentesis of the right knee reveals the presence of intracellular and extracellular weakly positive birefringent crystals in the synovial fluid. Gram stain is negative. Which of the following is the most likely diagnosis? A. Gout B. Septic arthritis C. Calcium oxalate deposition disease D. Reactive arthritis E. Pseudogout

E. Pseudogout is diagnosed by positive birefringent crystals.

56.2 Symmetric bilateral ulnar deviation of both hands in a 42-year-old woman A. Gonococcal arthritis B. Gout C. Pseudogout D. Osteoarthritis E. Rheumatoid arthritis F. Systemic lupus erythematosus

E. Rheumatoid arthritis gives the ulnar deviation of the fingers. *DIPs is NOT RA*

21.1 A previously healthy 18-year-old college freshman presents to the student health clinic complaining of pain on the dorsum of her left wrist and in her right ankle, fever, and a pustular rash on the extensor surfaces of both her forearms. She has mild swelling and erythema of her ankle, and pain on passive flexion of her wrist. Less than 1 mL of joint fluid is aspirated from her ankle, which shows 8000 polymorphonuclear (PMN) cells per high-power field (hpf) but no organisms on Gram stain. Which of the following is the best initial treatment? A. Indomethacin orally B. Intravenous ampicillin C. Colchicine orally D. Intraarticular prednisone E. Intravenous ceftriaxone

E. The patient described best fits the picture of disseminated gonococcal infection. She has the rash, which typically is located on extensor surfaces of distal extremities. Pain on passive flexion of her wrist indicates likely tenosynovitis of that area. The fluid is inflammatory, but gonococci are typically not seen on Gram stain. Ceftriaxone is the usual treatment of choice for gonococcal infection. Nafcillin would be useful for staphylococcal arthritis and would be the more likely choice if she were older, had some chronic joint disease such as rheumatoid arthritis, or were immunocompromised. **Consider also a single dose of 1200 azithromycin for chlamydia!!** (IM Ceftriaxone for Gonorrhea) Gonococcal arthritis is the most common cause of infectious arthritis in patients <40 years. Indomethacin or colchicine would be useful if she had a crystalline arthritis, but that is unlikely in this clinical picture. Intraarticular prednisone is contraindicated until infectious arthritis is ruled out.

41.2 A 40-year-old moderately obese woman presents with abdominal pain after eating and mild scleral icterus. Her laboratory results are AST 200 U/L, ALT 150 U/L, alkaline phosphatase 355 U/L, total bilirubin 3.5 mg/dL, direct bilirubin 1.8 mg/dL, and albumin 3.5 g/dL. A. Hemolysis B. Alcoholic hepatitis C. Gilbert disease D. Pancreatic cancer E. Gallstones F. Primary sclerosing cholangitis

E. The patient's laboratory results show a conjugated hyperbilirubinemia consistent with an obstructive pattern. She has the risk factors for gallstones (middle age, female, obese) and has symptoms of postprandial abdominal pain.

50.2 A 22-year-old college student with fever, headache, photophobia, and 25 white blood cells per high-power field but no red blood cells or xanthochromia in CSF A. Migraine headache B. Tension headache C. Cluster headache D. Subarachnoid hemorrhage E. Meningitis

E. The presence of white blood cells but no red blood cells in the CSF is indicative of meningeal inflammation, likely due to viral or bacterial infection.

empiric treatment of acute inflammatory diarrhea

Empiric treatment with quinolone antibiotics is usually indicated for acute inflammatory diarrhea. An exception is for enterohemorrhagic E coli (EHEC) infection, where antibiotics may increase the risk of HUS.

Timeframe of back pain resolution (if uncomplicated) ideal management

4-6 wks resolution in 90% of patients Analgesics, such as nonsteroidal anti-inflammatory drugs or narcotics, muscle relaxants, and attempts at maintaining some level of activity are helpful in managing acute low back pain; bed rest does not help.

Definition of Nephrotic Syndrome

>3.5g protein/24h hypoalbuminemia edema Often w/ HYPERcoaguability and hyperlipidemia (fatty/waxy casts)

3.1 A 28-year-old woman has been told she has rheumatic heart disease, specifically mitral stenosis. Which of the following murmurs is most likely present? A. Diastolic rumble at apex of the heart B. Early diastolic decrescendo at right-upper sternal border C. Holosystolic murmur at apex D. Late-peaking systolic murmur at right-upper sternal border

A. A diastolic rumble at the cardiac apex suggests mitral stenosis. - early diastolic decrescendo murmur is typical of aortic regurgitation, -holosystolic murmur at the apex is typical of mitral regurgitation -late-peaking systolic murmur at the upper sternal border is typical of aortic stenosis.

20.3 Which of the following is the best screening test for early diabetic nephropathy? A. Urine microalbuminuria B. Dipstick urinalysis C. Renal biopsy D. Fasting blood glucose E. Twenty-four-hour urine collection for creatinine clearance

A. Although a 24-hour urine collection for creatinine may be useful in assessing declining GFR, it is not the best screening test for the diagnosis of early diabetic nephropathy. In the outpatient setting, a dipstick urinalysis is readily available but will detect only patients with overt nephropathy (proteinuria >300 mg/d). Thus, a random urinary albumin/creatinine ratio of 30/300 is the best test to screen for early diabetic nephropathy. A fasting blood glucose may aid in the diagnosis of diabetes but not nephropathy. Finally, although most patients with nephrotic syndrome require a renal biopsy for diagnosis, a patient with worsening renal function who has had long-standing diabetes is assumed to have renal disease secondary to diabetic nephropathy, and the majority of these patients do not undergo a renal biopsy.

48.1 A 55-year-old accountant complains of facial and tongue swelling. He recently started using a new bath soap. His medical problems include osteoarthritis and hypertension, for which he takes acetaminophen and lisinopril, respectively. Which of the following is the most likely etiology? A. Lisinopril B. Soap hypersensitivity C. Hypothyroidism D. Acetaminophen E. Food-related allergy

A. Angiotensin-converting enzyme (ACE) inhibitors are often associated with angioedema.

2.1 A 55-year-old man is noted to have moderately severe congestive heart failure with impaired systolic function. Which of the following drugs would most likely lower his risk of mortality? A. Angiotensin-converting enzyme inhibitors B. Loop diuretics C. Digoxin D. Aspirin

A. Angiotensin-converting enzyme inhibitors and beta-blockers decrease the risk of mortality when used to treat CHF with impaired systolic function. For this reason, these agents are the initial choice to treat CHF. They both prevent and can even, in some circumstances, reverse the cardiac remodeling.

24.1 A 35-year-old obese hotel housekeeper presents with 1 week of lower back pain. Her history and examination are without "red flag" symptoms and completely normal, except for her weight. Which of the following is the best next step? A. Regular doses of a nonnarcotic analgesic B. Six weeks of bed rest C. MRI of the lumbar spine D. Plain film x-ray of lumbosacral spine

A. Bed rest has not been shown to improve outcome in idiopathic low back pain compared to encouraging usual activities that do not exacerbate the pain. Imaging is not necessary with uncomplicated back pain. "Red flags" = Unexplained weight loss, fever, hx of cancer that metastasizes to bone, IVDU, prolonged steroid use, age over 50. Bowel/bladder dysfunction, saddle anesthesia, decrease in sphincter tone and deficits at multiple levels can suggest cauda equina syndrome which requires emergent therapy.

36.2 A 62-year-old asymptomatic woman is noted to have multiple myeloma and hypercalcemia, but no bone lesions or end-organ damage. Which of the following therapies is useful for immediate treatment of the hypercalcemia? A. Bisphosphonates. B. Erythropoietin. C. Dexamethasone plus thalidomide. D. Interferon-α. E. Observe without treatment since she is asymptomatic.

A. Bisphosphonates are helpful in controlling hypercalcemia through inhibition of osteoclastic bone reabsorption. Dexamethasone, in combination with thalidomide, is useful in treatment of the myeloma, with a slower effect on the calcium level.

36.3 A 22-year-old African American woman presents with worsening cough and shortness of breath over 6 weeks, which did not improve with a course of antibiotics or antitussives. Her serum calcium level is found to be 12.5 mg/dL, and a chest x-ray reveals bilateral hilar lymphadenopathy. She has erythema nodosum on her legs. Which of the following is the most likely diagnosis? A. Sarcoidosis B. Mycoplasma pneumonia C. Acute lymphoblastic leukemia D. Squamous cell carcinoma of the lung E. Pulmonary embolism

A. Both sarcoidosis and lymphoma can present with cough, dyspnea, and hilar adenopathy on chest x-ray. In approximately 10% of cases, sarcoidosis can cause elevated calcium levels through the production of 1,25-vitamin D that occurs in the macrophages of the granulomas. This can also be seen in granulomas caused by tuberculosis and in lymphoma. Leukemia usually does not present in this manner, although it can cause hypercalcemia. Squamous cell carcinoma of the lung would be unusual in a patient of this age, and the radiographic presentation is atypical.

34.1 Which of the following are the most likely physical examination findings in a patient with emphysema? A. Diffuse expiratory wheezing B. Clubbing of the fingers C. Bibasilar inspiratory crackles with increased jugular venous pressure (JVP) D. Inspiratory stridor E. Third heart sound

A. COPD is characterized by chronic airway obstruction, with most airflow resistance occurring in small airways of the lower respiratory tract, producing expiratory wheezing. Inspiratory stridor would occur with upper airway, usually extrathoracic, obstruction. Clubbing is not generally a feature of COPD and should prompt investigation for another disease process such as a bronchogenic carcinoma. Crackles, elevated JVP, and an S3 are signs of congestive heart failure.

53.1 A 59-year-old woman is being seen for a health maintenance appointment. She has not seen a doctor for over 10 years. She had undergone a total hysterectomy for uterine fibroids 12 years ago. The patient takes supplemental calcium. The physician orders a fasting glucose level, lipid panel, mammogram, colonoscopy, and a Pap smear of the vaginal cuff. Which of the following statements is most accurate regarding the screening for this patient? A. The Pap smear of the vaginal cuff is unnecessary. B. In general, colon cancer screening should be initiated at age 60 but this patient has very sporadic care; therefore colonoscopy is reasonable. C. Because the patient takes supplemental calcium, a DEXA scan is not needed. D. Pneumococcal vaccination should be recommended.

A. Cervical cytology of the vaginal cuff is unnecessary when the hysterectomy was for benign indications (not cervical dysplasia or cervical cancer) and when there is no history of abnormal Pap smears. Colon cancer screening is generally started at age 50. DEXA scan for osteoporosis is recommended for women starting at age 65, or earlier for women with elevated fracture risk. Pneumococcal vaccine is generally given at age 65.

53.3 An 18-year-old woman is being seen for a health maintenance appointment. She has not had a Pap smear previously. She currently takes oral contraceptive pills. She began sexual intercourse 6 months previously. Which of the following statements is most accurate regarding health maintenance for this individual? A. A Pap smear should not be performed in this patient at this time. B. The HPV vaccine should be administered only if she has a history of genital warts. C. The most common cause of mortality for this patient would be suicide. D. Hepatitis C vaccination should be offered to this patient.

A. Cervical cytology should be deferred until age 21 (or 3 years after initiation of sexual intercourse.??) This is due to the fact that adolescents many times will clear the HPV infection and cause an abnormal Pap smear to normalize. The ACIP recommends that the HPV vaccine should be recommended to both males and females between the age of 9 and 26. The most common cause of mortality for adolescent females is motor vehicle accidents. The hepatitis C vaccine is currently not available, but hopefully in several years, it may be developed. In general: Screen every 3 years between 21 and 65 with Pap or combine with HPV and longer interval. Always depends on documentation of prior screening and sexual activity

56.4 Acute onset of unilateral elbow swelling, warmth, and tenderness and cervical discharge in a 25-year-old woman A. Gonococcal arthritis B. Gout C. Pseudogout D. Osteoarthritis E. Rheumatoid arthritis F. Systemic lupus erythematosus

A. Cervical discharge and inflammatory joint are consistent with gonococcal arthritis, which can also present as a migratory arthritis. Treat with IM ceftriaxone! If chlamydia needs to be treated - azithromycin

3.3 A third-year medical student has been reading about the dangers of excessive anticoagulation and bleeding potential. He reviews the charts of several patients with atrial fibrillation currently taking Coumadin. Which of the following patients is best suited to have anticoagulation discontinued? A. A 45-year-old man who has normal echocardiographic findings and no history of heart disease or hypertension, but a family history of hyperlipidemia B. A 62-year-old man with mild chronic hypertension and dilated left atrium, but normal ejection fraction C. A 75-year-old woman who is in good health except for a prior stroke, from which she has recovered nearly all function D. A 52-year-old man with orthopnea and paroxysmal nocturnal dyspnea

A. Clinical factors associated with a higher risk for embolic stroke include: -congestive heart failure -hypertension -age >75 -diabetes, or prior stroke. Echocardiographic factors include: - dilated left atrium or the presence of an atrial thrombus. The man in answer A has "lone atrial fibrillation" with a CHADS2 score <2, and has a low risk for stroke and thus would not benefit from anticoagulation. **use CHADSVAS (Vascular - previous MI, PVD or Aortic Plaque; Age 65-74, Sex (female) ) 0 score is "low" risk and may not require anticoagulation; a 1 score is "low-moderate" risk and should consider antiplatelet or anticoagulation, and score 2 or greater is "moderate-high" risk and should otherwise be an anticoagulation candidate. "Lone AF" = pt <60 yo w/o evidence of structural heart disease, HTN, or other factors for stroke. Anticoag w/ aspiring may be considered over warfarin...

56.1 Which of the following is most likely to be associated with advanced OA? A. Disability with recurrent falls and inability to live alone B. Joints with redness and effusion C. Best treated with oral steroids D. Improvement throughout the day after approximately 1 to 2 hours of "unfreezing the joint"

A. Degenerative joint disease is a major cause of decreased functional status in elderly patients and requires ongoing treatment and evaluation by the physician to try to improve symptoms and to promote mobility. Oral steroids are not helpful in this condition.

15.3 A 74-year-old man with no prior medical problems faints while shaving. He has a quick recovery and has no neurologic deficits. His blood sugar level is normal, and ECG shows a normal sinus rhythm. Which of the following is the most useful diagnostic test of his probable condition? A. Carotid massage B. Echocardiogram C. Computed tomographic (CT) scan of head D. Serial cardiac enzymes

A. He likely has carotid hypersensitivity; thus, careful carotid massage (after auscultation to ensure no bruits are present) may be given in an attempt to reproduce the symptoms.

33.3 A 23-year-old man is found to have late latent syphilis (RPR 1:64) as part of a workup following his diagnosis with HIV. He is asymptomatic with a CD4 count of 150 and does not remember having lesions or rashes in the past. Prior to starting therapy with penicillin for the syphilis, the patient should undergo which of the following procedures? A. Lumbar puncture to exclude neurosyphilis B. Skin biopsy to confirm the diagnosis of syphilis C. Magnetic resonance imaging (MRI) of his brain and an electroencephalogram (EEG) D. Skin testing to exclude penicillin allergy E. Adjustment of his HIV medications to optimize his CD4 count prior to treatment for syphilis

A. Lumbar puncture to exclude neurosyphilis is generally indicated when any patient with syphilis develops neurologic or ocular symptoms, or if HIV-infected patients with syphilis have a CD4 less than 350 or an RPR more than 1:32.

51.1 Which of the following patients is most likely to be a candidate for bone mineral density screening? A. A 65-year-old, thin, white woman who smokes and is 15 years postmenopausal B. A 40-year-old white woman who exercises daily and still menstruates C. A healthy 75-year-old white man who is sedentary D. A 60-year-old overweight African American woman E. A 35-year-old asthmatic woman who took prednisone 40 mg/d for a 2-week course 1 week ago

A. Of the choices, this woman is the only individual with risk factors. Risk factors include white race, age, postmenopausal status, smoking, positive family history, poor nutritional status, and chronic treatment with a drug known to predispose to bone loss.

4.4 Which one of the following patients should be promptly referred for endoscopy? A. A 65-year-old man with new onset of epigastric pain and weight loss B. A 32-year-old patient whose symptoms are not relieved with ranitidine C. A 29-year-old H pylori-positive patient with dyspeptic symptoms D. A 49-year-old woman with intermittent right-upper quadrant pain following meals

A. Patient in answer A has "red fl ag" symptoms: he is older than 45 years and has new-onset symptoms. Patient in answer B may benefit from the reassurance of a negative endoscopic examination. Patient in answer C, however, may benefit from treatment of H pylori first. Some studies indicate this approach may be cost-saving overall. This patient could be sent for an endoscopic examination if he does not improve following the therapy.

51.3 A 60-year-old woman presents with the results of her DEXA scan. She has a T score of −1.5 SD at the hip and −2.5 at the spine. Which of the following is the most accurate interpretation of these results? A. She has osteoporosis at the spine and osteopenia at the hip. B. She has osteoporosis in both areas. C. This is a normal examination. D. She has osteoporosis of the hip and osteopenia at the spine. E. You need to know the Z score.

A. The T score is the number of standard deviations of a patient's bone mineral density from the mean of young, adult, white women. It is the standard measurement of bone mineral density used by the World Health Organization. A score of −2.5 SD is the definition of osteoporosis. A Z score is the number of standard deviations from the mean bone mineral density of women in the same age group as the patient.

7.1 A 32-year-old woman with a 5-year history of HIV infection is noted to have a CD4 count of 100 cells/mm3. She is admitted to the hospital with a 2-week history of fever, shortness of breath, and a dry cough. Which of the following diagnostic tests would most likely confirm the diagnosis? A. Silver stain of the sputum B. Gram stain of the sputum showing gram-positive diplococci C. Acid-fast smear of the sputum D. Serum cryptococcal antigen

A. The fever, dry cough, and dyspnea are consistent with PCP, which is diagnosed by silver stain of the sputum, which often requires bronchoalveolar lavage to obtain. At CD4<200, pt should be on TMP/SMX, Dapson, or Atovaquone for prophylaxis against PCP

59.3 A 75-year-old woman, diagnosed with stage 0 CLL 1 year ago and being monitored without treatment, now complains of fatigue and dyspnea. She has no palpable adenopathy or splenomegaly, no rashes or arthritis, and her CBC shows ALC 11 000/μL, with hemoglobin 6.8 mg/dL, and platelet count 127 000/μL. What is the most appropriate diagnostic test? A. Direct antiglobulin (Coombs) test B. Antinuclear antibody C. Bone marrow biopsy D. Test for Lewis alloantibody

A. The most likely diagnosis is autoimmune hemolytic anemia (AIHA), which can be confirmed by detection of antibody and/or complement components on the surface of the RBC, usually by the direct antiglobulin (Coombs) test. AIHA is a common complication of CLL. ANA to screen for systemic lupus erythematosus has a low probability in a woman of this age, without other clinical features of SLE. Bone marrow biopsy to evaluate for bone marrow failure due to CLL could be considered, but rapid progression to stage III/ IV would be unlikely. Lewis alloantibodies have no clinical significance.

35.3 A 22-year-old African American woman presents with fatigue, arthralgias, and a nagging dry cough for the past 6 weeks, but no shortness of breath. On physical examination, her lungs are clear to auscultation, and she has bilateral pretibial tender erythematous raised nodules. Which of the following is your best next step? A. Chest radiograph B. High-resolution CT C. Empiric treatment for postnasal drip D. Antinuclear antibody E. Initiation of antituberculosis therapy

A. The patient has clinical features suggestive of sarcoidosis given the new cough, arthralgias, and description of erythema nodosum. The initial, most cost-effective study is a chest radiograph. Hilar lymphadenopathy with or without interstitial infiltrates would solidify a diagnosis of sarcoidosis. A high-resolution CT may be ordered if the patient has interstitial lung disease, but it is not the first study of choice. Postnasal drip does not explain the patient's other symptoms. An antinuclear antibody would not necessarily identify the cause of the cough or provide a diagnosis

50.3 A 31-year-old woman with a long history of intermittent severe unilateral throbbing headache lasting hours to days associated with nausea and photophobia, but no preceding symptoms and no visual disturbance, occurring once or twice per month A. Migraine headache B. Tension headache C. Cluster headache D. Subarachnoid hemorrhage E. Meningitis

A. The patient's history is strongly suggestive of migraine, given its unilateral and throbbing character, and the associated symptoms of nausea or photophobia. Most patients with disabling headache have migraine. Tension headache should have none of these features.

51.2 During which of the following periods in a woman's life is the most bone mass accumulated? A. Ages 15-25 B. Ages 25-35 C. Ages 35-45 D. Ages 45-55

A. The time of greatest accumulation of bone mass in women is during adolescence.

9.3 A 34-year-old woman is noted to be diagnosed with stage I hypertension and after an evaluation is noted to have no complications. According to JNC 7, which of the following antihypertensive agents are generally considered firstline agents for this individual? A. Thiazide diuretics B. Angiotensin-receptor blockers C. Alpha-blocking agents D. Nitrates E. Vasodilators such as hydralazine

A. Thiazide diuretics such as hydrochlorothiazide or chlorthalidone are generally considered first-line agents for uncomplicated hypertension because of their effect in reducing cardiovascular mortality and their cost-effectiveness. **Thiazides have been shown to decrease mortality; esp in African americans! - 1st choice for Isolated Systolic HTN in elderly). CCBs have better effects in blacks If pt has CKD, consider ACEi or ARB

22.3 A 17-year-old sexually active adolescent boy presents with a 5-day history of fever, chills, and persistent left ankle pain and swelling. On physical examination, maculopapular and pustular skin lesions are noted on the trunk and extremities. He denies any symptoms of genitourinary tract infection. Synovial fluid analysis is most likely to show which of the following? A. WBCs 75 000/mm3 with 95% polymorphonuclear leukocytes B. RBCs 100 000/mm3, WBCs 1000/mm3 C. WBCs 48 000/mm3 with 80% lymphocytes D. WBCs 500/mm3 with 25% polymorphonuclear leukocytes

A. This history is suggestive of gonococcal arthritis, and the rash is suggestive of disseminated gonococcal disease. The synovial fl uid would most likely show an acute inflammatory exudate, WBCs 72,000/mm3 with 75% polymorphonuclear cells.

60.2 A 75-year-old man presents to the emergency room with the sudden onset of nausea and vomiting. His medical history is notable for coronary artery disease and well-controlled hypertension. On examination he refuses to open his eyes or move his head, but when finally coaxed to sit up, he immediately starts to retch and vomit. Rotational nystagmus is noted. He cannot walk because of the dizziness and nausea that walking evokes. His noncontrast brain CT scan is read as normal for age. Which of the following is the best next step? A. MRI/magnetic resonance angiography (MRA). B. Obtain a thorough psychosocial history. C. Dix-Hallpike maneuver. D. Prescribe meclizine. E. Referral to neurology.

A. This patient has symptoms of central vertigo. The onset of symptoms was abrupt and severe. His gait is affected. If he were able to cooperate with an examination of his cerebellar functions, it would most likely be abnormal. His age and history of hypertension and coronary artery disease place him at elevated risk for cerebellar infarction or hemorrhage. CT is not the appropriate test for examining the brainstem; MRI is much more accurate. MRA may be useful for delineating the exact vascular cause of the symptoms.

4.3 A 45-year-old man was brought to the ER after vomiting bright red blood. He has a blood pressure of 88/46 mm Hg and heart rate of 120 bpm. Which of the following is the best next step? A. Intravenous fluid resuscitation and preparation for a transfusion B. Administration of a proton-pump inhibitor C. Guaiac test of the stool D. Treatment for H pylori

A. This patient is hemodynamically unstable with hypotension and tachycardia as a consequence of the acute blood loss. Volume resuscitation, immediately with crystalloid or colloid solution, followed by blood transfusion, if necessary, is the initial step to prevent irreversible shock and death. Later, after stabilization, acid suppression and H pylori treatment might be useful to heal an ulcer, if one is present.

15.4 A 49-year-old man is admitted to the intensive care unit (ICU) with a diagnosis of an inferior myocardial infarction. His heart rate is 35 bpm and blood pressure 90/50 mm Hg. His ECG shows a Mobitz type I heart block. Which of the following is the best next step? A. Atropine B. Transvenous pacer C. Lidocaine D. Observation

A. This patient's bradycardia is severe, probably a result of the inferior myocardial infarction. Atropine is the agent of choice in this situation. Mobitz type I block has a good prognosis (vs complete heart block), so transvenous pacing is not usually required.

11.1 A 25-year-old medical student is stuck with a hollow needle during a procedure performed on a patient known to have hepatitis B and C viral infection, but who is HIV negative. The student's baseline laboratory studies include serology: HBsAg negative, anti-HBsAb positive, anti-HBc IgG negative. Which of the following regarding this medical student's hepatitis status is true? A. Prior vaccination with hepatitis B vaccine B. Acute infection with hepatitis B virus C. Prior infection with hepatitis B virus D. The student was vaccinated for hepatitis B but is not immune

A. This student's serology is most consistent with vaccination and not prior infection. Like all health-care workers, the student should have been vaccinated against the hepatitis B virus, which induces anti-HBs IgG antibody, which is thought to be protective. Not all people receiving the vaccine develop an adequate antibody titer; if none were detected, it would indicate the need for revaccination. Patients with prior hepatitis B infection will also likely have anti-HBsAb but will also have anti-HBc IgG. *Acute infection would be signified by the presence of either HBsAg or anti-HBc IgM.

47.3 A 67-year-old woman with extensive atherosclerotic cerebrovascular disease complains of dizziness and vertigo. Which of the following arteries is most likely to be affected? A. Vertebrobasilar B. Carotid C. Aorta D. Middle cerebral

A. Vertigo and dizziness can be seen in vertebrobasilar insuffi ciency.

2.4 A 55-year-old man is noted to have congestive heart failure and states that he is comfortable at rest but becomes dyspneic even with walking to the bathroom. On echocardiography, he is noted to have an ejection fraction of 47%. Which of the following is the more accurate description of this patient's condition? A. Diastolic dysfunction B. Systolic dysfunction C. Dilated cardiomyopathy D. Pericardial disease

A. When the ejection fraction exceeds 40%, there is likely diastolic dysfunction, with stiff ventricles. The stiff thickened ventricles do not accept blood very readily. This patient has symptoms with mild exertion that are indicative of functional class III. The worst class is level IV, manifested as symptoms at rest or with minimal exertion. ACE inhibitors are important agents in patients with diastolic dysfunction.

13.5 A 32-year-old man presents to your clinic with Kayser-Fleischer rings, dysarthria, and spasticity. Pick the cause from the following that is probably responsible for the patient's presentation. A. Wilson disease B. Hematochromatosis C. Primary biliary cirrhosis D. Sclerosing cholangitis E. Autoimmune hepatitis F. Alcohol-induced hepatitis G. Viral hepatitis

A. Wilson disease is an inherited disorder of copper metabolism. The inability to excrete excess copper leads to deposition of the mineral in the liver, brain, and other organs. Patients can present with fulminant hepatitis, acute nonfulminant hepatitis, or cirrhosis, or with bizarre behavioral changes as a result of neurologic damage. Kayser-Fleischer rings develop when copper is released from the liver and deposits in Descemet membrane of the cornea.

Glycemic Goals for T2DM

A1C <7.0% preprandial glucose 70-130 mg/dL, or postprandial glucose <180 mg/dL. Blood pressure should be <130/80, and LDL cholesterol should be <100 mg/dL.

MC cause of sepsis in older pts

Urinary tract infections and pneumonia

FEV1 vs. FEV1/FVC in obstructive and restrictive processes

Whereas in both obstructive and restrictive lung disease, the FEV1 is decreased, the FEV1/FVC is decreased in obstructive processes and normal in restrictive processes.

Initial pharm management of OA

acetaminophen. Joint replacement for severe osteoarthritis is reserved for patients with intractable pain despite medical therapy and for those with severe functional limitations

Peptic Ulcer Treatment

acid suppression with an H2 blocker or proton-pump inhibitor to heal the ulcer, as well as antibiotic therapy of Helicobacter pylori infection, if present, to prevent recurrence.

Indication for Cardiac resynchronization therapy (CRT) and placement of an implantable cardioverter defibrillator (ICD) can

advanced heart failure and low ejection fraction <35%.

definition of microalbuminuria

albumin secretion 30-300 mg/d if present, treat w/ ACEi or ARB even if pt is normotensive **If pt is african american, consider CCB, which will protect against cardiovascular events and reduce microalbuminuria

Progression of Aortic Stenosis

angina, exertional syncope, and heart failure

Syphilis Treatment

based on stage: early syphilis can be treated with a single intramuscular injection of penicillin; late latent syphilis can be treated with three weekly injections; and neurosyphilis or tertiary syphilis can be treated with intravenous penicillin for 10 to 14 days.

classic triad of multiple myeloma

bone pain due to lytic lesions, anemia, and renal insufficiency.

Central causes of vertigo

cerebellar hemorrhage or infarction, can be immediately life-threatening and require urgent investigation.

Acute onset of dyspnea or hypoxemia with a normal chest x-ray

considered a pulmonary embolism until proven otherwise

management of healthy women with acute uncomplicated cystitis

cultures are not routinely sent, and treatment can be initiated based on symptoms and on a urine dipstick finding of leukocyte esterase or nitrites Treatment = 3 days of : TMP/SMX, ciprofloxacin (fluoroquinolone) or nitrofurantoin

A-Fib Management Unstable vs. Stable

direct current cardioversion if the patient is unstable. If the patient is stable, initial management is ventricular rate control with an atrioventricular nodal-blocking agent, such as beta-blockers, diltiazem, or verapamil.

Syphilic chancres

generally clean, painless, ulcerative lesions and can be located anywhere on the body where inoculation occurred.

Symptoms of malabsorption

greasy, voluminous stools, weight loss, anemia, neurologic disorders from deficiencies of B vitamins, and osteopenia from deficiency of vitamin D and calcium.

Most Common causes of A-Fib

hypertension, atherosclerotic heart disease, pericardial or pulmonary disease, and hyperthyroidism.

Auscultatory findings in mitral stenosis

loud S1 and an opening snap (OS) following the sound of second heart sound (S2). The interval between S2 and OS varies inversely with the severity of the stenosis.

major cause of morbidity and mortality in patients with type 2 diabetes mellitus

macrovascular disease, such as coronary artery disease, stroke, and peripheral vascular disease, so aggressive cardiovascular risk factor reduction is essential.

Common sites of OA

most often affecting the distal interphalangeal joints > proximal interphalangeal joints > knees > hip joints.

major side effects of statins

myopathy and hepatocellular injury.

management of most acute infectious diarrhea

oral rehydration solution or with antimotility agents such as loperamide. Most cases of acute infectious diarrhea in the US cause mild to moderate illness that is self-limited,

typical x-ray finding in rheumatoid arthritis

periarticular bone erosion (loss of joint space)—may not develop until later in the disease process, when the diagnosis has already been made based on clinical findings.

Platelet level at which spontaneous hemorrhage may occur

platelet counts of less than 10 000/mm3.

MC Causes of chronic cough in immunocompetent nonsmokers who are not taking ACE inhibitors

postnasal drip, asthma gastroesophageal reflux disease.

UTI Diagnosis

presence of urinary symptoms AND > 10^5 colony-forming units (CFUs)/mL in a clean-catch specimen and more than 10^2 CFU/mL in a catheterized specimen.

Gonococcal arthritis - other symptoms

presents as a migratory tenosynovitis, often involving the wrists and hands, with few vesiculopustular skin lesions. Treatment = IM Ceftriaxone, oral dose of azithromycin for chlamydia

symptoms of suspected celiac...and workup

relatively mild gastrointestinal symptoms, and may only present with unexplained nutritional deficiency (eg, refractory iron deficiency anemia). If there is a high clinical suspicion for celiac disease, patients should undergo endoscopic evaluation with small bowel biopsy and serologies for IgA anti-endomysial antibodies and anti-tissue transglutaminase (TTG) antibodies. typical malabsorptive symptoms with foul smelling diarrhea to subclinical disease. It should be the differential diagnosis of iron deficiency (especially in men but also in woman who have menstrual losses but do not respond to iron therapy); metabolic bone disease (usually patients are tested for this disease especially if osteomalacia but also osteoporosis without typical risk factors; abnormal liver enzymes as before; and a peripheral neuropathy.

in absence of trauma, mc cause of acute monoarthritis

septic or crystalline arthritis

lab values of patients w/ osteoporosis

should have normal serum calcium, phosphorus, and alkaline phosphatase levels. Laboratory abnormalities should prompt a search for an alternative diagnosis.

68-year-old female sees her physician for tiredness and fatigue. Evaluation reveals she ishypothyroid where TSH is 42 (nl 0.4 - 4.5) free T4 0.4 (nl 0.8 - 2.7). The patient has no other medicalproblems. dose of synthroid and when to decide euthyroid

start with low dose 25ug because you want to decease chance of increasing HR/Contractility -> angina/ischemia Check TSH every 6 wks after starting/changing dosing (T4 has a long half life).

Presentation of PCP CXR and LAb findings

subacute presentation with fever and a dry cough, almost always in patients with a CD4 count less than 200 cells/mm3. Patients may have normal chest x-ray or a faint bilateral infiltrate ypically have an elevated serum lactic acid dehydrogenase level.

Presentation of acute leukemias

symptoms due to symptomatic anemia, bleeding due to thrombocytopenia, infection due to neutropenia, or with hyperleukocytosis and symptoms of CNS or pulmonary microvascular ischemia.

Pts w/ a LDL goal <70

those with established coronary heart disease (CHD), and other atherosclerotic vascular disease, such as stroke or peripheral vascular disease, or diabetes, which is considered a "CHD equivalent." The low-density lipoprotein goal for very-high-risk patients may be less than 70 mg/dL.

Pain profile of OA and treatment goal

worsened with activity and is not associated with morning stiffness. No pharmacologic agents that modify or stop disease progression are available. Treatment is aimed at symptom relief.

41.4 A 32-year-old man with a 5-year history of episodic bloody diarrhea and abdominal cramping pain presents with scleral icterus and fever. His laboratory results are AST 100 U/L, ALT 125 U/L, alkaline phosphatase 550 U/L, total bilirubin 5.5 mg/dL, direct bilirubin 3.0 mg/dL, and albumin 2.9 g/dL. A. Hemolysis B. Alcoholic hepatitis C. Gilbert disease D. Pancreatic cancer E. Gallstones F. Primary sclerosing cholangitis

F . The patient's laboratory results show a conjugated hyperbilirubinemia with an obstructive pattern. The history is consistent with inflammatory bowel disease, which is associated with primary sclerosing cholangitis. The initial evaluation should include ultrasonography to rule out gallstones; if negative, ERCP could confirm the diagnosis by demonstrating multiple strictures of the extrahepatic bile ducts. Treatment options include stenting of the larger bile duct strictures and immunosuppression to slow the progression of the disease

most common cause of thyrotoxicosis

Graves disease. No other diagnosis is likely if the patient has bilateral proptosis and a goiter.

MC Causes of duodenal and gastric ulcers

Helicobacter pylori infection and use of nonsteroidal anti-inflammatory drugs.

MC cause of acute symptomatic vs. asymptomatic hypercalcemia

Hypercalcemia that is acutely symptomatic is most likely caused by cancer. -Expect elevated Ca and suppressed PTH Asymptomatic hypercalcemia is most likely caused by primary hyperparathyroidism. -expect elevated PTH, Ca with decreased Phosphate levels

Digits involved in Rheumatoid arthritis

In nearly all patients with rheumatoid arthritis, the wrist, metacarpophalangeal joints, and proximal interphalangeal joints are affected, whereas the distal interphalangeal joints are spared. Distal interphalangeal joints and large weight-bearing joints are most commonly involved in osteoarthritis.

CML vs. Leukamoid reaction

Low leukocyte alkaline phosphatase (LAP) and presence of basophilia are seen in CML, and help distinguish it from leukemoid reaction (high LAP).

Cyrstals in Gout vs. CPPD

Monosodium urate crystals in gout are needle-shaped and negatively birefringent (yellow) under the polarizing microscope. Calcium pyrophosphate dihydrate crystals in pseudogout are rhomboid and positively birefringent (blue).

Treatment of gout depends on the stage:

NSAIDs, specifically indomethacin colchicine, or steroids for an acute gouty arthritis, urate lowering with probenecid or allopurinol during the intercritical period.

MC cause of nongonococcal septic arthritis

Nongonococcal septic arthritis is most often caused by S aureus and most often affects large weight-bearing joints.

definition of osteoporosis

Osteoporosis is defined as a T score of −2.5 SD. score is the number of standard deviations of a patient's bone mineral density from the mean of young, adult, white women. DEXA in women > 65, men > 70 regardless of risk factors

back pain symptoms suggestive of infectious/neoplastic causes

Pain that interferes with sleep, significant unintentional weight loss, or fever

MC Cause of secondary HTN

Renovascular HTN

only medical therapies shown to decrease mortality among persons with chronic obstructive pulmonary disease.

Smoking cessation and supplemental oxygen to treat chronic hypoxemia


संबंधित स्टडी सेट्स

Chapter 21: Genomes, Proteomes, and Bioinformatics

View Set

NMT exam symptoms: Sciatica, low back pain, knee pain, plantar fasciitis, hip pain, ankle pain

View Set

Maternal newborn practice test B with rationale

View Set

History honors - The renaissance

View Set

Quiz on approximations and irrational numbers

View Set

A Personal Mission Statement and Vision

View Set